You are on page 1of 674
Copyright © 2006 by Firewall Media (An Imprint of Laxmi Publications Pvt. Ltd.). All rights reserved. No part of this publication may be reproduced or distributed in any form or by any means, or stored in a database or retrieval system, without the prior written permission of the publisher. International Standard Book Number : 81-7008-933-6 987654321 Limits of liability and disclaimer of warranty : The information and material contained in this book are provided “as is", without any warranty concerning the accuracy, adequacy, or completeness of such information or material or the results to be obtained from using such information or material. Firewal] Media (An Imprint of Laxmi Publications Pvt. Ltd.) shall not be responsible for any claims attributable to errors, omissions, or other inaccuracies in the information or material contained in this book, and in no event shall Firewall Media (An Imprint of Laxmi Publications Pvt. Ltd.) be liable for direct, indirect, special, incidental, or consequential damages arising out of the use of such information or material. Published by FIREWALL MEDIA Branches (An Imprint of Laxmi Publications Put. Ltd.) Bangalore (Phone : 080-26 61 15 61) 22, Golden House, Daryaganj, Chennai (Phone : 044-24 34 47 26) New Dethi-110002 Cochin (Phone : 0484-239 70 04) India Guwahati (Phones : 0361-254 3669, 251 38 81) Phone: 011-23 25 25 74 Hyderabad (Phone : 040-24 75 02 47) Fax: 011-23 25 9572 Jalandhar City (Phone ; 0181-222 12 72) Kolkata (Phones : 033-22 27 37 73, 22 27 52.47) Lucknow (Phone : 0522-220 95 78) Mumbai (Phones : 022-24 91 54 15, 24 92 78 69) Ranchi (Phone : 0651-230 77 64) http/www Jaxmipublications.com: All Rights Reserved with the Publishers. {This book or part thereof cannot be reproduced or translated into any other language without the written permission of the Author and the Publishers.] FIN-2856-396-INTEGRAL CALCULUS MADE EASY First Edition : 2006 Price : Rs. 395.00 Only. (C=11337/05/12 Typewetring by : Goswami Printers, Delhi-110053. Printed at : Sanjeev Offset Press, New Delhi. 1 Fundamental Integration Formulae 1.1 INTRODUCTION Integration is the inverse process of differentiation. In differentiation, we are given a function and we are required to find its derivative or differential co-efficient. In integration, the derivative of some function is given and we are required to find that function. 1.2 ANTIDERIVATIVE OR PRIMITIVE A function F(x) is said to be the antiderivative of the function f(x) on the interval (a, 5) if a @& FPel=Aa), Vee la, d). And we write d a Fel = fx) - J rena: - Fw. SOME SOLVED EXAMPLES ‘Example 1. Find the antiderivative of the function fx) = <*. 4 Solution. From the definition of antiderivative it follows that the function F(x) = = is an antiderivative because dafx* a3) oa a. Example 2. 7 (sin x) = cor x Solution, feosxds = sinx ‘Also, ifc is any constant, Then 2 (sin x + ¢) = cos x. ‘Therefore ;In general [ cos xd = sin x+c. 1 2 INTEGRAL CALCULUS MADE EASY It shows that different values of c will give different integrals and hence a given func- tion may have an indefinite number of integrals. Remark. The presence of indefinite constant c justifies the name Indefinite Integral. ‘Thus, we may conclude that Z [Fe] = ix) = J fends =F) +6 where c + constant and known as constant of integration. Note (i) The symbol | is elongated S, and represents the summation and stands for integral of the given function, i) flx), the function which is to be integrated, is called the integrand. i) J dx is called integral w.r.t. x. (iv) ¢ is called constant of integration and it means any real number. (o) xin | fix) dx stands for variable of integration, (vi) The integral of a function may or may not exist. In other words, it may not be possible to find a function whose derivative is equal to the given function. (vii) The integral of a function if it exists is not unique and any two integrals of a function differ constant, (viii) The geometrical meaning assigned to the integral is area of some region. (ix) Integral is used to find physieal quantities like contre of mass, momentum ete. gt Result 1. Prove that | x" de =~ 46, when n +—1. aD mst a a Sol. We have 2 #"")_@+Dx" z dz|nvi} @+D nal ‘Thus, we may write tea Belo, The above Result is also known as Power Formula. Note. The Power Formula, could be memorise as inerease the power ofx by one and divide by the increased power. Result 2. Prove that [ 2de = lag | x1 +0, where x20. Sol. Clearly citherz>0 or x <0. FUNDAMENTAL INTEGRATION FORMULAE » Therefore, there may be two cases : Case I. Whenx>0,then |xle=x a ad 1 dx Hoe! x I= 5. (log x) = — Therefore, in this case, we get. 1 J Sax = log I xl +e. Case IL. When x < 0, then | x | =—x ad d 1 1 dx [log I x I] = dx Hog (-* = ca CO PRts Therefore, in this case, we get J ds =logilxl+ec Thus, from both the cases, we have J de =log Ix I +e. 1.3 SOME STANDARD ELEMENTARY INTEGRALS Based on differentiation and definition of integration, we have the following standard results. The student is strongly advised to commit these results to memory, because no further progress is otherwise possible. (i) <(@)=0 => [ 0.ax =c od ; (ii) a=) = f ledz exec (iii) (he) =k = [ kde skx +e n+ ww 22 1 ine Det axtine-l dx|n+1 ~(ntD kn _ = f= -dx =" +e;n#-1 d 1 1 (v) Glog |x l= — = J Se slog ixi+e (ut) fe) =e => J ef dx mete, cin £{ 2) oes =a*;a>0;a41 z = [ of de= c +cj;a>0,a41 log a INTEGRAL CALCULUS MADE EASY (witty (ein x) = cos x => | cosxdx = sin x +e (ix) © 008 x) = sin x = [ sinx de =~ cosx +e (x) © (tan x) =» sec? x = | sec? xr =tanz+e dx (xi) © (cot x) = cosec* x => | cosec? x dx =-cotx +e crit) (see x) = see x tan x => | secxtan xdr = seex+¢ (xiii) £ (— cosee x) = cosce x cot # => | cosec x cot x dx =—ecosec x +0 (xiv) a (sin! x)= i => | ! dx =sin"' x +c dx 1- x" ji-x? a 4.).—2 1 — tan-l (rv) = (tan »)=7, 3 => J se stan xr+e . ad 1 1 (xvi) —(sec"! x) = => a ir = sec! x +e dx x¥x"-1 JES d 1 Note. We know that ——(- cos"! xj = de i-x? From this result, it should not be concluded that sin-! x = - cos-! x. Rather, sin-! x and cos! x differ by a constant. (xvii) 4 (cos! x) = —— => f at de = 008 x40 dx 1-x? 1-x? .», @ ‘ -1 -1 = = dx = cat"! (xviii) x (cot-* x) ext = f lex? cot x+c da om | -1 (xix) — (cosec"! x) = => dx = cosec™! x +c. dx xe? -1 J zyx? -j 1.4 INTEGRATION OF ALGEBRAIC, LOGARITHMIC, EXPONENTIAL AND RATIONAL FUNCTIONS Example. Evaluate the following integrals : a I ae 5* oon | x (i) Jj cw (ii) J dx (iii) J a (iv) [ x4 de (v) fyeu (vi) [e?™* dx x FUNDAMENTAL INTEGRATION FORMULAE (vii) j e™ dx (viii) fase dx (x) [ Yb? ap. Solution. (i) J dx = [x3 de x x a = +c 1 -~=+1 3. a5 =F teed Mee 3 . » (ii) fo dese te 1 (iii) | sam de = x ¥? de (34) +c Cr -12 say tes-arM@ere 2 (+1) a =| = x (iv) x” dx KésD +c x? =-—+te 5 1 1 -th —dx=[ —de = | x dx (u) lz ah 1 a-l fan) LS “(Ea (2=4) +c= ft (vi) Jerse idx = [e®*"dx = [xd (ix) fia ~ . _ ght : J v den +6] ts m log n = log n™] [‘ elt = fix)] 6 INTEGRAL CALCULUS MADE EASY (ui) fer dx =ze4+e. ° (viii) fab * de =f ab&* de [ve al * =x] dal 4 = {x". de = ~~ 4-2 * = |x = 301 +c= 4 +c r aX = a ‘ a * dy = a* (ix) 5 -dx i dx | Ja joga +e| af 2 ~— #1 p | (x) J Ye? -dp = ftp?) ap = fp ap = 4 fe |: Jt de= 2 +6] ~+ 3 63 3 = Fe +e= sp se. 3 1.5 IMPORTANT EXTENSIONS OF ELEMENTARY FORMS (a) All the results mentioned in the standard elementary integrals holds good when x is replaced by (x + a) in any formula. where a — any constant. For example : . ( + yred (i) [xtay" dx = TF +0; (n #— 1) a (x+a)"*! -~ 1 (n+) (e+ay" = (x + a)". dzx| (n+1) (n+ 1) 1 ae —— dr — . _ (it) xta =log |! x+al+e;x4-a. (iti) J sec’ (ix+a).dzx =tan(x+a)4+e (iv) fertt*.dr wert +e, (6) If x be replaced by (ax + 6) on both sides of any standard result mentioned in the list, the standard form remains true, provided the result on R.H.S. is divided by ‘a’ ie., the coeffi- cient of x, where a and 6 being constants. For example : (ax +6)"*! . by" dxr= (i) | (ax+ ) (na l)a +c; (nz-1) FUNDAMENTAL INTEGRATION FORMULAE, «(n+ 1) (ax +b. dx| (n+).a | (nt).a 1 fy te ax+b a Ses. 1 w (ai) foce*(ar +b) de= fan taced) 4e exh tiv) few? ae = +e. 4.6 THEOREMS ON INTEGRATION Theorem L. If ffx) be an integrable function on x, then Proof, Let us consider that [fia de = ga+0 By definition Sigel = fiz) = Steals 20 =fs)+0= fx) Elfreode] =p. a =(ar+ b) Efron AD oul) [By using equation (1)] Tv By using equation (2)] Note. This result shows that integration and differentiation are inverse processes and they neutralize the effect of each other. Theorem 2. Jrio ax =a] pivas where k =» constant. ie, the integral of the product of a constant and a function is equal to the product of constant and the integral of the function. Proof, Let us consider that ficode =a) ‘Then, by the definition of an integral, we have d ax B= fz) = 2 (eet) = ke. Sige =k. fx) = Joe fa) de =k gta = Je Perde=k. [radde Al) (2) Iv By using equation (1)] {+ By definition of an integral] [+ [rtarde= ats)] 8 INTEGRAL CALCULUS MADE EASY Remark. If fix) = 1, then Je-denk. frdeak[2°de-teoe, ‘This implies that, the integration of a constant 4 with respect to x is dx. ‘Theorem 8. Prove that the indefinite integral of algebraic sum of two or more functions is equal to the algebraic sum of their integrals. ie, Jota + geod = [fied + faced. Proof. Let us consider that Jris).de =A AD) and Jato.az =a) 2) £ yen=n0 8) and tata = ae Aa) = Symsae= Linon Zee =f) + ete) Jofer+ e0o-dr = Aad+ gi) (By definition of an integral] By using equations (3) and (4)] ef fiside+ [glade [+ By using equations (1) and (2)) Remark. (i) The generalization of the above result is fino fala) +... + fx@O). de = Jrwadcs fpirdstus fhladde. Gi) It can be easily derived that the indefinite integral of the difference of two functions is equal to the difference of their integrals. (iii) The results of theorems 2 and 8 can be generalized to the form flficers hafae) nn lode = inf flzres + inf aerae cot fey J falco ie, The integration of the linear combination of a finite number of functions is equal to the linear combination of their integrals. 4.7 IMPORTANT EXTENSION FORMULAE OF STANDARD INTEGRAL FORMS nat @ forge +e; (n#-1) et = Jeers by dea Gt sere _leglartdl «a fo -de tog tet +e = laa * = FUNDAMENTAL INTEGRATION FORMULAE 9 z= (iii) fat .de=— 4 03(a>0,a4- log a mirth => jam? de= oe tes (a>0;a#21) mirth e (iv) Jet -ae =e4+e = Jem"? .de= +c (v) foosx.dx =sinx +e = fos (ax +6) de = Siniar +6) be (vi) [sin x dx =-cosx +e = fein (ax +b)de =-8E*) (vii) [scc? x de =tang+c = [ec? (ax +b) dx = SEE *) +c (viii) [ cosec? x dx =-ctxr¢e¢ = Jose? (ax +b) dx = — ota +6) +e a (ix) [seex tan x dx =secx+c = foce (ax + b) tan (ax +) dx = 82 (ae*?) +¢ (x) [cose x cot x dx =-—cosecxr+c cosec (ax + 5) -——|a de SOME SOLVED EXAMPLES Example 1. Evaluate the following integrals : => [cose (ax +b) cot (ax +6) dx = (i) fa-xVz dex (ii) [Vr de (iii) | (e? -3x+ 4) dx (iv) [ a* .e* de (v) | 2.3% de (vi) [ (x? - 2e+ 4)? dex (vit) f x4 de (viii) | gt? de (ix) f 37° dx Solution. (i) [ G-2 Ve de =f We-xVx)de = f x? de-f x? de n+¢l1 ts fot ae n+1 10 INTEGRAL CALCULUS MADE EASY x 2 2 - ten gmt x +e. Pr dn Va+1 ntl “ _f wa x “ " dy = (ii) | Yeax=f x dx = j +¢c [= ax ote —+4] 3 = Set +c. 2 ‘ a ttl (iit) [x -82+4)de = [x dx-3[xde+4f de : [x dr=——— +e 3 2 = 5-- 7S parte. (ae)* a f * * dx = * = a r = (iv) fa e J (ae)* ax log(ae) *° : fe dx oe a" e* = log (ae) *“ 2y 9X E gi z we x - a* (vp) [ 2% .a%dx = [ 4*.3%de=[ 12* de : fa dx Ze] 12" = jog 12 * (vi) | (x? - 2244)" de [ (a+b +e)? =a? +b? +c + 2ab + he + 2ca] = | (x4 + 4x7 + 16 - 4x9 - 16x 4+ 8x") de - | (x4 — 4x9 + 12? - 16x + 16) dz =| x4 de- a] x* de +12] x* de ~ 16{ xdx+16{ dx a+] : [ xtae== +e| n+l] 5 4 a 2 x 4x x x eS tPy - By + lOx +e xo xt + 49 — Be? + lO 4c. on] FUNDAMENTAL INTEGRATION FORMULAE weet ned (vii) M4 de== te as fxtaee® J Ba fxta © dee OT =01( 25) + {orden 2 +6] (ix) Jat dra fa" de [em log n= log n™) an =Jeueti te te a" oa] aise Example 2. Evaluate the following integrals : i) f geet ay flow @ f sFar id fF see xdz 3 (itt | JEG? + 28? -2 + 9)de ww [(¥-4) dz [ faracn io ai [feet xae=3 fcc? rdr =F tne ee, (iit) [ Vela? +20? 5 +8) de = fata? +20? -243)de = fet 4 22 2? 43x") de = fa deo afc de fa de +8 fede , 12 INTEGRAL CALCULUS MADE EASY (io) S(4-4) a= fler- a (a-6)' =a— 68 Baba - 6) = fl? 9 32 Pe ar = fee ae ede = foe? =a ~ 30" 432) dx = fs dx— fe deaf x dz tafe" de 3 1 1 we) g 2) gy! gat @) estanfé 2 fede fort ide= Fo - ~241*°= Example 8. Evaluate the following integrals : of(eD(e dle Go JE ae tin (50) ax toy [50° 4204-104 feo 2) ae ( [EEE ae. FUNDAMENTAL INTEGRATION FORMULAE 1\(.2, 1 Solution. (i) f(z+2}{« + las zx x = fl? +3} {2 +5) la = i= thee 3 late : f(# ta4ie 5 Jae = fetes [x.de+ [ae + fax git} lel ‘el wit = 301 + Tat eglal+ = 341 +c 4 2 <2 x = x fo + | lx l4+——+#e “a 2° 8 -2 4 2 x" x 1 = —+— +log | x | — —— +e 4° 9 78 Se iy JER ae [+B Sere florea) =afl.dr+b[tde+e x? dx =x “241 cz =ax+blog|x!l+—— 77 801 +C “1 cr =ax+blogixl+— 7 +C€ sax+blogix!-—+C. (iii) J fe+5=* Jae= farm d+ [5x* dx = 3fx«” dx +5{x* dx hea 3x 2 Batt? 2 #c23.—. x84 “Ty 401 1 +e Z =6 x +x 4c. (iv) is + 2x76 tat fo 9) a = a de +2[ x de—1[x.dr+ [= de+5{—dx a4+1 =i+1 L+i -¥2+1 = ox 42% 2s += +5loglzl+¢e 3+1 -5+1 1+1 lui 2 4 4 x"? =e, re +=] +5log zi +e 2 14 INTEGRAL CALCULUS MADE EASY eae Ta aie l+x+x° (l+x)+x" (1+x) x (v) dy = Jar (lea) x*(1+x) il; 7+) ag fie +f a= iE der ft lez gael Se slog (142 ten > a tog 1142 | ee =~ = + log | lex! +e. Example 4. Evaluate the following integrals : (i) Joss dx (ii) fa ~ 42°) dx -x° 4+4- ! ix 2 (iti) (re) = ja [rz dx (v) Jaca ‘ de. 1 +1 2 Solution. (i) [Vax+0 de = [lax +b)” de = eet te Fs v2 _ x+y c=— 2 (ax + b)*? + ¢. Ge) ™ 3° (ii) [e4x? 4x) dx =4fx® de—4f x dx 4,3*1 4y75t1 4x4 4x “Gal -ea*¢" a a te 1 ext+—7 +e ; . 2 ~ giex?*+x-1 eee - -1 (x-Di(x" +) =a7(x -1)41(x -1) (iii) eae Tem ee —— fr x(x x J x-1 J (x-D =(x —1)(x? +1) 3 = |(x?+Ddr = [ x? . Je Jz dx +f dx 3 +x+C (iv) J x? dx = l+x*-1 5. [Add and subtract 1 to the numerator] 1+x? 1+ x? 1+x* 1 = fl Sept = fae] are -tan2x+c : [chp aestant xed +x FUNDAMENTAL INTEGRATION FORMULAE 15 wf @-3* Vaden f Wt-62+9). lade = fa? 62 4.9).2 tem fx 6x 4 92) dr = f 2 dx —6f2%* de + 9f 2" de. 3 1 att get"! get! a7? ae? ont? Bh Shr Ta Wa 82 a2 2 2 2 <224 2 60% Example 5. Evaluate the following integrals : ( J et +Se0na— 40! +.2)dx tip f (- 20+ aa f on wy ( 22 ae wf FS io) f EE x? yf x42 ofa wo fF a Bx ty [ et? (oid J oe (wits) [ See ax. Solution, (3) fle! +3 conx—4x* + 2)ds af farsa] cosxde-4f a2 dx+2] de ‘ wersasinz- “2 s20e0 se+3sinz-x'+2r+c. io { (a 1 aege-of taeaf Wo f(e ~2e" +B)ae= Je de 2f et dee f Aue oo + fetde= +e = net tog let te 1 a J deslogizl+e. z = Jet et 4 log lel te 2 x ad tes Since the degree of numerator is greater than the degree of denominator, therefore by actual division, we have 16 Is l+x lix- Loe -Jre- fa [Add and subtract 1 to the numerator of second intergral] 1 =f ede-J aes] O net v J tden2 0 nel =x+log | 1+x 1 +c. J dae=togict+e x 1 . qez, _ f lare Gu) eae J x41 1 “Jae {Add and subtract 1 to the numerator of the second integral] 1 esto J “+s Tee og | 14x 1-x4logl14xi4e log | 1x 1-240. wes ae Since the degree of numerator is greater than the degree of denominator, therefore by actual division, we have : 3 x4 ix? —4( x- ota [= » Ale * aux . 1 @+bP =a? +b? + 2ab) |e 18 INTEGRAL CALCULUS MADE EASY ‘engl 0 Bealaate the Slowing ines: w f(E+4 +fast eat +a). dx ;a>0 w@] omy, de Gi) f eet 4 cote 4 c0te), de toffee - Solution. () S(E+de= +0" +03) a2 fedrea flare fae det fat devo [x.de Z 57+} ©) fie" +0" +e" 0% +sina).de. 1 - x dx 2 = Es etogisls +e. loga pet set Gi) Jott d= isaa* sit fam de x oa oo: * Shoge *° : Je de = Egg ia Oia al (Gi) fle? + eth 4 et OtA) de = Jeetee 4 eh 4 ede te mlogn =log n™) = flat +2° +0%).dr [ee 5 fa] “4 +@*. x4. = fot.des fet. der fat dx “tga ani wo ff s-7F -+}.e x fucace | iypae~ 2] Fg ae Joa a weetante—Qeirte- 3 46 : J ggg tertante reef Ppa wsin"ts +e FUNDAMENTAL INTEGRATION FORMULAE 19 ©) Jl? 407 +e at 4sin a). de = Je ade Jat de [cay d+ fina. de zo) ot (eal - tiga aes Tine zte Example 7. Evaluate the following integrals : » f= ay [+O idx weer faq a [TE ae aay f atte? ey - x See tay amar. win [ wf nT A Solution. () [7 —.de x +l ‘ = oe [Add and subtract 1 to the numerator] : a®~b? =(a +bXa-b) (2? - x7 +) 1 — det dx x+l Ira » (xt 1) (x9)? 17 2(e? -127 0) =Jot-vde fore sae fidesf rede = 2 -estarizee [- Jpdze-tntard| saane ti {TERY ae @ 2 [OP HO +20" .b* ate -f [<= OF, 20h) a*b* a*b® a*b’ (i @+ bP =a? +206 + BY 20 INTEGRAL CALCULUS MADE EASY watt (x +2074 1-2? @-24) (e240? 2? (t-a+) * (cP 414 20x? 41-2) (a? =x +1) -dx [Add and subtract x? to the numerator) dee = fet e+ D) de Tv (e?-64 =(a- Xa +8] sfx? dre fx.dx+] dees Bare, fo 1 de (Add and subtract 1 to the numerator] Ji+z * . 2 © See “(HE- sea) de = files)? de- fia sy dx “ase?! aia?” eee Gt? Gea “fi -1 3/2 v2 (Fea). aja = F408 2040! 4e, Example 8, Evaluate the following integrals : . 1 o 2x | eae ©) pees (ii) f xYSe=2 dx Gi) f (a4 eet). 1 i = de Solution.) Let I laa | L Jour i- faxt2 senalizati |) sates eee * [On rationalization] - ee 1- \2z42 (@a+)-(ax+2) =f Se af Wev8 de- J fev ide =f x42)" de~f ar+ 0” de . ge = (ae BT [: Jlaxrbt de eat? +c L (a+6)(a-6)=a?-B*] FUNDAMENTAL INTEGRATION FORMULAE 21 day dot x42)?" Grey? a tt aoe) 2(3+3) (ax + 2! — Fae UM +0 = glide + 2) Qe+ IPA] +e. f 2x (i) [| —2* ede | eS =| BE x NOPE INOUE ay [On rationalization] Yats+ja-x yars-Ja-= [Te @+5)(a-6)=0?~8%) =| EE ae | are | te =f @s+n arf @-n? ae tay Lat (a +x)? (a~ x)” a py _ fax BT . (3-3) ots)” [: J exe or ae EO 2 2 2 2 = Farr + Fa-n +0 2 = 5 lesa Ha- Ml +6, (iii) Let 1s f x/5e-Bax of Sx or=3 as (Multiply and divided by 5) - zJ (5x -242)/5e=2 de [Add and subtract 2} =3J (x—-2)f5x—2 d+ 2 {5x-B dx ati -9” 2 a = Ef 6-2 ara Z | e-2)" de INTEGRAL CALCULUS MADE EASY _ 1 Gr- git 92a a? e 8 ye oe by** [: J tas oy" ar= aD --| 2 4 = Fog Gt BF + ae Ge— 2 a0. io) f 8 Hebd bes a fak-tare | ear e Jot de= to re “foe ferttaee Example 9. Zualuate the following integrals : @ Jeg, x-de lees ide 2 Gi FY ae © | Fostee of a ade 0 fat Solution. (i) flog.x.de=J1.de = +e. (slog, .a = 1) [eee Jat Ee (On rationalization} (a? = 6) = (a + ba -5)) = j ete) Grta~Je+b) 4 te “Gta)-G+d) ” agate a-& fast ae elk eta 1 ey! a-6 32 o-6 32 *° FUNDAMENTAL INTEGRATION FORMULAE 23 1" e+ o ae 5g arb ee 3 a 2 +b)? +e -b 3° a-b ~~ ® 2 iti J Beh ae. ‘Note. If the integrand is a rational function whose numerator and denominator are polynomials and degree of numerator is greater than or equal to that of the denominator, then, first divide the numerator by the denominator and then use the result : F = Quotient + ae] Denominator. (ox4 0? dx? 4441 x-2) at yazed les 2 —. de = | ————.. da -Bx x-2 x-2 + 26 1ax+1 = fae de 412224 2%, x=2" act =p Wee 25 log | x- 21 +c met + 1Br + 25 log | x-2 1 ee. Wo |e cr a ze =| 1 V8 +8- V62=2) oe (On rationalization] Noa+8+ o-2 “isd fore = _ f vGe43 - J5x-2 (gt B= _ = erp ly @-B6)=(@ + ba -8)1 =f cen ft te [Ta] =p [Jox+9" ae fex-2" ax] 1 Pees | “5) 3 3 3° 25 =1f2 pa _ 2 og, gyn §[ Zee +99? - 2 x-a29?] 6 = Gre 9 —Gr- B14 6. 24 INTEGRAL CALCULUS MADE EASY [ie mlog, n= log, n™] =f elt ("2 = Fay] ©) [ate * yxtl-yx-2 1 Jx+1+Jx-2 ‘i, enti: 2 . (On tionalizatic See = (On tionaintioy 2 fein 2 (> @*=04) =(a+ bXa—6)) (e+ D-(2-! a = pe? = Efe vY? des fie-2" dr] 1 [G+ | @-2)” 1f2 ao, 2 972 opr a sex 3[Baew ge +c 2 22 eer Dee 2h te, Example 10. Evaluate the following integrals : yy feetl a) [24 fate Go J 25 -a. Solution. First, see note given in the part (éi) of above example. o faye) she “S(* -s?414—2 )e x41 = fct-2?+D.de 25 xe) Fat let ety FUNDAMENTAL INTEGRATION FORMULAE. = fet de- fa? des fide wet x xt - - et-Derte — 5 8 a=xt41 axt-g? + + xed el Example 11. If f(«) = ty Solution, We have fa) = [r'@).de = J (5 1 1 “Je loe > fix) = log tx 1 + tan x4c x when x = 1, f) => FT elog 1+ tan! 1+ Fr0+E rem c=0 Aix) =log |x + tan tx, 1) 26 INTEGRAL CALCULUS MADE EASY Example 12. [ff (x) = a sin x + bcos x and ro=4,=3,7(2 = 5, find ftw. Solution. We have fid= Jrvo.de = J(asin x+5 008 2).de za fsinzdr+d [cos x.dr = fit)=-acosx+bsine+e (1) when x =0,/(0)=4 = 4=asin0+bcos0 = 42046 = bed (2) whenx = 0, 0) =3 By using equation (1) = 3=-acos0+bsinO+e = Bs-ate = e-a=3 (8) whens 4,7(E) = 2 2 By using equation (1) Epa k = 5=-acos F+bsin = +e = B= -al0) +b) +e 2 5=bec = Bade [By using equation (2)] > c=1 4) ‘Substituting the vlaue of c in equation (3), we have = 1-a=3 = as-2 Now, substituting the values of a, b and c in equation (1), we have fix) =2cosx+4sinx +1. 1.8 INTEGRATION OF TRIGONOMETRIC AND INVERSE TRIGONOMETRIC FUNC- TIONS Example 18. Evaluate the following integrals : (i) Jean? x.de Gi | f= sin Be ode Gi) [tin x +008 x).de Gv) [ cosee x (cosee x + cot x) dx FUNDAMENTAL INTEGRATION FORMULAE 27 Solution. (i) [tan? xdv = fisee? x- Dae [++ eect A- tan? A= 1] = Jove? ede fide =tanz—x4e. (i) f JIB Be ae = f (eos? x + sin? 29-2 sin x c08 x)” de [+ cos? A+ sin? A = 1, sin 2A = 2sin A cos A] = [llcos x sin a)*}"* de = (cos x sin x) dx = Joos xd~ fain xd = xin s—(-c08 x) +6 =sinx +cosx+c. (iti) (sin +008 x).de~ [sin x de+ [eos de cosx+sinx +e. (iv) Joosee x (cosec x + cot x)de cosec® xdx + fcosec x cot x.dx cot x — cosec x +c. in 57 SRE |. ae cos? x cos? x = Jevec? see xtan x). de = Joc? x.dde— free xtan x.de stan x—seex te. Example 14. Evaluate the following integrals : @ Je see” x —7 eosee® x + 30" )dx wf tan(3z - 5) sec (3z - 5) dz iy fein? oy fein? titi) | sin® 3 ae (iv) | sin? 2x4 Dade () J cost Beds. Solution. () { (sec? x-7 cose? x4 3e*) dx =6f sect xde~Tf cosec? x de +3) et dx =6 tanx —7(—cot x) + Se" +e = Gtanx +7 cotx+ Set +c. wi) J tan (82 ~ 6) sec(3z ~5) dz => 1-cosA=2sin? — 2 =e oe, [« J secs tan 2 dx =uee x +e] (ii) J sin? ga = 3f 2sin? 5 dx [Multiply and divided by 2] 1 ss 1-eos 2A =2sin? A = 5] (-cosarde ‘ 28 INTEGRAL CALCULUS MADE EASY 1 1 =3J ldr-3 cos xdx 1 . 1 . = pep tins te= 5 e-sinal+c. tiv) f sin? ax+ Dae -f $sin ax + 1)- Zain tae + 0] a 4 4 > sin3A=3sin A—4sin® A = 4sin® A=3sin A-sin3A = sint A=Ssin A-1 sin 3A 4 sin (e+ Dde-2f sin 6x+3)de asset) [==] SS | - =| Eee 2 4 6 = $ cos (2+ 104 Gt cos(Gr + 8)+e. () J cost ax dx =f (cos? 20)? de 1+ cos 4x =J drests) dx ‘+ 1+008 2A = 2e08? A = 1+c0s 4A =2cos? 2A (142 cos 4x +008? 4x) dx > J J (1+2c08 tes tesB ae 4 i 4 (14200842444 eons) ax (§ +ecears 2 eons) de te ies kl lee # ax+3f vos dx des 2 cos Bx dx (2) (a) +a | |] 2\ 4 8. 8 FUNDAMENTAL INTEGRATION FORMULAE 29 Example 15, Evaluate the following integrals : sin? x 1 ade of Teens as Wi [sro (ie) [sin Ji veon Be wd 243 cos (wy f 22 Sees ain? x ~ cos? Solution, ) f sin’ x ge [ 1008" = gy [: sin? A+ cos? A = 1) 1+cos x Te cos x tens 2-08 x) 4 = Jo1-cos x) ax T+ 608 x = fide Joos rd ax—sin x +e. 7 {2 sinx «@ [per [e a? b? (a4 bXa-B)) sinx cos” [v sin? A + cos? A = 1] = Jee? x de — Jace xtan x de = tan x ~s0ex +0. [Multiply and divided by 4] wre ames =4]—— dt =4 Poy de lar Jrsee * (> sin 2A =2 sin Acos Al At cot 2x) 2 —2cot 2 +c. (ivy [sin x, fT+c08 Be ce = sin x. 2 008% x de cos 2A = 2 cos? A - 1) = V8 fsin.x.cos ede = F [asin coon de {Multiply and divided by 21 sp Jsin2eae (> sin 2A.=2sin Acos A] 30 INTEGRAL CALCULUS MADE EASY + 2430082 2 Seos x w Je a(S. se 2). as 2 or Sin? = =2 fonsee! x de-+8 fot x cosee x dx =—2eotx—3 coseca +c, Example 16. Integrate the following functions : (i) fan x + eat xP (gi) £08.28 = 008 2a, cos x = cos a (iii) z (iv) (2 tan x - 3 cot x)? , ©) ~. sin? x cos” x Solution. (i) {tan x +cot x)" dx = fetan? x + cot? 2+ 2tan x cot 2). de [sect A— tan? A= 1, cosee? A cot? A = 1) af {eect x1 comet «14 2.4an 2.4) ode tans = Jicoe? x + cosce? 2-242). = [soe x + cosce? x) de = [sec? x. dx + [oosec? x.dy =tanx—cot x +0. sy { SOBRE = 0082 2 @w cosx-cosa” [cos 2A = 2 cos? A- 1) 2» )—(2eo8? 2 e— 2.008? (Boost xD Geoe a) gg . | Pom seme gy cia cos x— oso (cos? x - cos" a) (cos x +08 (eos x cos ct) 2 cerccsa cas c—eosa “ot af (cos x — cos a) de =2 f(cosx cosa). dr =2 feosx.de+2 feosa.de 22 feos xdr+2c0s 0: [1.de =2sinx +2reosa+e. sin? x cos? (sec? x~ Wd = [sec x.dx- [1dr =tanz—x +0. sec? A -tan? A= 1) Sede = [tant 2dr FUNDAMENTAL INTEGRATION FORMULAE 31 (iv) J(2tan x — Scot 2)? dz = Jretan 3)? + (3 cot x)? - 22 tan xX(3 cot x)].de = [G4 tan? x +9 cot? x 12 tan xeot 2).ds = Jtatsec? x — 0+ 9e0see" x - 1) - 12}. de [: tanzeotx=tan 2. =] tans = [ld sec? x- 449 cosee? x 9-12) de = [td sect x+ 9 cosec! 1-25) .ds a4 occ! xde+9 feoseet x.dx-26 fide =4 tan x9 cot x 25x40. = (gittng-qaitite,) «= (atest « fin? x oss in? xeon? = cont JarktieJask« = [seca tan x de ~ [ cosse x cot x dx = see x + cosee r+. Example 17. Evaluate the following integrals : eos? x- sin? x Seosx+4 ® de iy [EEF ae 0 ea 0 Pe io z 445 .in x (ii) [ Jt+sin& ae vy | de w) | fim sing dx wif aie ae « oi eet tan®” cos? x- cos 2x Solution. (i) == yee x= [ oe cos 2x {2-cos® 2x 1 ot agen te 60s x ‘| cosx +4 i) See ide sin’ x 2x sin? x 32 INTEGRAL CALCULUS MADE EASY Tie 4 ee" SEE de + 4 Poasect x dx af : sina‘ sinx 23 feosee reot x.dx +4 feosee? rae =-Becosee x—4 cotx +e. ws = iit) J jirsin 5 de + eos? A+sin? A=1 = J yoos? $+ sin? + 25in cos = oeke sin 2A =2sin A.cos A 4 4 aoa = sin A =2sin 5 con 5 “J fowS ssn) dee [foo + sin). ae (sin x/4) | (cos x/4) = Se ke V4 Va Joe = Joos Fax fin Fae te= Asin 4 cos 4 iv) [Ae 5sin st x deaf 4 des fo SP ae cos? x cos x ' cos x 4 = + (ao Se) = 4 face! x.dx+5 free xtanx.de =4tanx +5 see +c. w J ising de cos" A+sin? A=1 = Joos? Sain? 2-205 sin = de sin 2A =2sin A cos A \ 2 2 2 2 s ~A A > sin A =2sin “> cos > FUNDAMENTAL INTEGRATION FORMULAE 33 | sin? (ei) lees? a+ cane = sin A=2sin cos | vnactunbons| 2 2 * 1+c08 2A = 2 cos* A = 14608 A= 20s? 4 tan? = dx Ee sec? A-tan® A = 1] _ (eect —tan x) de [On rationalization] (see x - tan x) tye = f Se ets ay [se a®=b' = (a= bXa4 d)] sec* x—tan* x = five! x= see x tan x) de Ie sect A=tan® A = 1] = foect x.dx— Jseex.tan ide =tanx-secx+c. Example 18. Evaluate the following : of =a cose Gi) [sin x08 x (sin 2x + 008 22). de 1-cosx se tan x 1- cos 2x id | Sian a do) f (E8832) ts = - 2 ope Solution, (i) { 8—SE2E yp = f SO eae x= Dy 1-cos x l-cosx x Boos? x+1toosx yf -(2cos* x-cosx- 1) 1- cos j= = (cos xD) (20s x + 1Xeos x - 1) = f Sess Mess ae = [@cosx+D.dr (cos x- 1) =2 foosx.de+ [1dr =2sinz+xee. INTEGRAL CALCULUS MADE EASY (i) [sin x 00s x (sin 2x + cos 2x). de = 3 fe sin x cos x). (sin 2x + cos 2x).dx [Multiply and divided by 2] sin 2A = 2 sin Acos A} = 3 Jin 2x (in 2x + cos 2x}, dx u 3 fitsin? 2+ sin 2x cos 2x). de = + Je2sin® 2x+2sin 2x-c0s 22) ae [Multiply and divided by 2 again] = 1 f1-cos + sin 4x) x [is 1-cos 2A =2 sin? Al =} [Jt-ae— feos dx.ax + Join dx dx] 1/, _sinde _ cos de 4 4 Gii) Please try yourself. (Ans. see x—tanx+x4e] 1-cos 2A =2sin? A’ 1+ cos 2A = 2cos? A stanx-xte Example 19. Evaluate the following integrals : ° offs + sin 2). sect x. de (iy J ee sin? xeos? x 7 sin® x , i cin | ee w [rae wo {(e ~ cos: 2 Sar sotation (a f= 285), see de 2 + (1 - cos? wf 2) nets Iv sin? A+ cos?A = 1) =f (PS ESE*) sets . 1+2 FUNDAMENTAL INTEGRATION FORMULAE 35 = f| TEE, see? 2-98 coo? x] de lex? +x” = Jove? x. de dx =tanx—tan e+e. ci [in ztomt z+ooa' x (a? +59) = (a+ 6)? = Sab(a +b) sin? x cos” x (sin? x + cos? x) -3 (sin? x + cos” x) in? 2 vax sin? x.cos® x sin? A + cos? A = 1] “Slgrtan-t}-« in? x + cos? = [SH dra fide sin? A+ cos* A = 1] sin? xeoe? x cos” x = + [$24 dea fide Vint reas?» J [> G?=6?)= (a+ bXa-d)} 1=2sin? x cos* x [Add and subtract 2 sin? x cos? x} = [Seinen n= 2sin® x cos? x}. (sin x + cos? xXsin? x~cos? 0] 4 1-2sin? xcos? x [y sin? A+ cos® A= 1] INTEGRAL CALCULUS MADE EASY = fisin® x 60s” x).de [ir cos 24 = cos? A~ sin? A] (Cs cos 2A = cos? A — sin? A) [Iv cos* A+ sin? A = 1] cos 2A =2cos* A-1 = ee h= 2 008? Ay 1 = Trg lessinal+e 2 st 2 et? loi, 2y 1 1, = 57g 7 Bta g Sine t= gx - Gx 5 sina te. FUNDAMENTAL INTEGRATION FORMULAE 37 1.9 EVALUATION OF Jain «dx AND Joos" ax Inorder to evaluate [ sin® xx and j cos” xdr for n= 2, 9, 4 the integrands sin* x and cos" x are expressed in terms of sines and cosines of multiples of by using the following formulae : (i) cos 2A —2sin?A ii) cos 2A = 2 cos? A= 1 (ii) sin 3A =3 sin A-4 sin’ A (iv) cos 3A = 4 cos’ A - 3 cos A. Example 20. Evaluate the following integrals : (i) J cos? ne ae (i) f cost eae Wii) [ sin! xe téo) f sin? ede, 1 Solution, (i) f cos mdz = 5] Bees ne dx [Multiply and divided by 2] 25 J (1+0m ne) de [ ¢0s 24 = 208? A= 1] x1 sin2nx “alt dea 3 J eos2ae. dr 4 te ied. sin 2nx +¢ (ii) J cont x.dx = f (cos? x)? dx 2 =| (PS ds = 2 (1008? 2x-+200828) dr 4 =1f (+ Tee. noose) de 4 2 1 1a ea) (103+ Seow du-+2con2) cs 1p (3 d “q ($+ Jeoe4x + 2c082) de “Sf a Jeon d+ 3 cos 23) dx =Bfrdes Tf onstr.de se} contxde _3e,1sindx 1 sin2e “eta 4 “272 *° neh sindes sin dee ity J sin® x.de w [ (sin® 2)? de Iv 005 2A = 1-2 sin? A} “SEPS) a =i) (1444554 acon) 7 J (1 re08? e208 2s 38 INTEGRAL CALCULUS MADE EASY 1 (143+ Beosax - 2eos2r) dx (3 +dcosax- Beast). dx J 4 1 *4 3 8 j J (+ Wnts tents} dk J Loads + 3 f cosas. de - jf em2eae 1sindx 1 sin 2x *e 04 2B sD singe— tein 2x +e ‘y sin 3A =3sin A-4ein A ivy f sin’ x.dx =f (Gain tsinax) ae = Asin? A =Bsin A—sin A 4 a a, 3.4 1a = sin’ A=3sin A—bsin A Example 21. Boaluste the following integrals : ( J sin? 2x4 5).dx Gi) J sin® x cos? x dx Citi) J 009? xed, Solution. (i) { sin? (2e+5).dx > cos2A=1-2sin? A = 2sin® A=1—cos 24 1-cos 2A 2 1 2(2e +65) fas = sin?A= 1 Lids ~ 5 J cos (4x +10). dx sin (4x + 10) ‘+e 4 sin (4x + 10) +c. 2 i 8 z_ 2 (i) J sin® x cost dx = f (sin x e08 2)* dy 1 {Multiply and divided by 16] a 1 f isin 2x) 2. f (sin e008 2). = 5g J (sine) ede J (sin? 2ay? de FUNDAMENTAL INTEGRATION FORMULAE. 39 1 ¢ (1 cosdx *: cos 2A =1-2sin? A * 16 * => 2sin? A =1—cos2A = sin? Aa £7008 2A 2 = sin? 24 = 1 c0s4A 2 1 2 = gq J atone 4x - 2008 42).dx -a/ (1+ 429889 ons) ae 1 1, cos8x =a) (urd . ~ Bean te).de 15 (8, cos8x -al (3-25 — neosas). as 1 = yyy) (9+ 000 8x —4 e004). de 1 =i J vedo f cos sx dx—4 f cos 4x.ds] = ylte+dene 424], - galt Jin —sin 4] 9 iii) J cos? xa = f (2) te a feoste.dr+f Scosx.dr |x cosA =4 cos? A-Beos A 4 = 4.0008 A=cosSA +3008 A cost A= 20838 +308 A 4 1.40 EVALUATION OF {sinmx cosnx d Jeosmx cos nx dx AND Joimmnx sin nx dx In order to evaluate [sin ms cos nx de, fcosmecasnede, [sin mesin neds, the integrands sin mz cos nx, cos mx cos ny, sin me sin nx are expressed in terms of the sum (or difference) of sines and cosines of multiples of x by using the formulae : (@) 2 sin A cos B = sin (A + B) + sin (A- B) Gi) 2.cos A sin B = sin (A + B)—sin (A - B) (iii) 2. cos A cos B = cos (A + B) + cos (A -B) (iv) 2.8in A sin B = cos (A — B)— cos (A + B) 40 INTEGRAL CALCULUS MADE EASY In applying these formulae, it is advisable to have A > B, because A > B would imply A-B>0. Example 22. Evaluate the following integrals : @ feos mx cosins de + when (a) m+n (b) m =n. Gi) finms sins de men > m-n#0. Solution. (#) (a) It is given that, when m # n, feos ms cos ned = 2 [2c08 mx cos nx de (Multiply and divided by 2] = 2 feos (a+ mx + cos m ~ nsx [++ 2.008 A cos B= cos (A + B) + cos (A ~ B)} 1 1 = Jeostm+ ma de-+ 5 [costn -n)x.dx _sin(m + nde, sin (m—n)r “Am +n) 2Am—-n) (6) When m =n ; cos mx cos nx dx = cos” nx dx J J = 3p cos? nx = 2 fas cos2nv).de [Multiply and divided by 21 1 1 2g) ede t 5 foot 2ne.de gh sindat 41 cin one ee. 2 2° Qn 2 4n° ~ wf nima sin nx di = 2 [2sin msin ne x [Multiply and divided by 2] 7 } Jleostin —ne~ cost + nix Ide (: 2sin A sin B= cos (A -B)— cos (A +B)] 1 1 = GJ costin—n) x.cde—5 fas (m+n) x.de sin(m=n)x _ 1 sin(m+a)x (m-n) 20 m+n in (m =n jae in(m+n)x (n-ne) m+n Example 23. Evaluate the following integrals : @ Join de sin 20 de (ii) Joos a cos x .dx (iii) [sin 4 008 3x dx (io) f sin 9s cas dx de. Solution, (i) [sin dx sin 2x dx f2sinaesin 2x dx [Multiply and divided by 21 FUNDAMENTAL INTEGRATION FORMULAE 41 = feos (8s 2x) cos x +2eNlde [+ 2sin Asin B = cos (A- B)-—cos (A+ B)] = j feos. ~ eos 52 de =f foos.x. de—F feos Sede essing - 2808 yal gine ginse re, = ene 2 5 “2 10 (i) Joos.4x cos x dx = 3 f Rens 4x 008 x de (Multiply and divided by 21 = 3 feos (4x + 2) +c08 alae Ls 20s A.cos B = cos (A + B) + cos (A~ B)) 1 1 1 = 5 J (co8 x + e08 Se) ds = gp Joosbxde + 5 feos dx AsinSx 1 sin’: sin5x | sin3x ape tg ge tee at ge te (iii) [sin 4. cos 3x dx = 3 f2sin 4xcos 3x de (Multiply and divided by 2) = 2 flsin (4s +32) +sin 4x -Seildr (+ 2. sin Acos B = sin (A+ B) + sin (A= B)) ley. * Ir. 1p. = 5 Jin te +sin ade = 5 fain Ta ds + 9 Join ede =1(-oet 2 7 cos 72 _ cosx 14 2 1 }* gb eosah te +e Go) [rin Bx cos dx ds - F fe sin 8 cos dx de [Multiply and divided by 21 = 5 fisin ax + An) +5in Gx ~ Aside (+ 2sin Acos B =sin (A+ B)+ sin (A~B)] sinxdr (> sin (— 6) = — sin 6] 42 INTEGRAL CALCULUS MADE EASY Example 24. Evaluate the following integrals : (i) [sin® x cos? x de tii) Joos x cos 2x cos 3x dx Ui) [sin x 6in 2e sin duds ivy [22 ae cos 2x (v) jae de. sin x Solution. (i) j 2 x cos" x dx = f(cin xcos x)" dx = 3 Jeesin xc0s 2)* dx [Multiply and divided by 8] =) fesin 239° = 5 fsin as dy Apis = pein! 2ede y sin3A=Ssin A -dsin® A = 4sin° A=3sinA -sin3A 1j(dea zen win’ a -38A sin BA es | er 4 8 4 . . > sin? gq =28in 2A -sin 6A 4 LT re, “te = E[Josin zeae J sin 6x cx] . 1 —Bece2e _(_ so6s) “32 2 6 1 3 1 = fl-Zemar+Zemar] +e. (ii) feos x ¢08 2x cos 3.x de = af cos x 008 2x).008 3x dx (Multiply and divided by 2} = 3 Jos 2x c00 xens de de = } feos 2e-+ 2) + ens 2x a3} cositx de [. 2cos A cos B = cos (A + B) + cos (A -B)] = 3 fico 9x +c09 2) 608 See = J feces? s+ cos 3x 008 2) ae = jf feeeos? 8x + 2e0s 8x08 x) de [Multiply and divided by 2 again] FUNDAMENTAL INTEGRATION FORMULAE 43 1 1 =f J 1+ 008 Geddes +3 [loos (x +x) +008 (x —xI)dx cos 2A =2cos* A-1 = 2cos* A=14 cos 2A = 200s? 3A = 1+ cos6A [+ 2cos A cos B = cas (A + B) + cos (A— Bj] i <2[y inte 4 Joins sin 2x sin 3x de = 3je sin 2x sin x)sin 3x dx (Multiply and divided by 2} = 3 floes (2 ~ 2) ~ cos (2x +x)).sin 3x dz [+ 2sin Asin B= cos (A-B) —cos (A + B)] 1 9 = 5 | tooo — coo 8x). sin dds = § fisinas cos ~ sin ax cosBx)de 1 [2sin 3xcos 3rd (Multiply and divided by 2 again} = 3 fisint2 +2) + sin @x ~ xihde ~ +t [sin Gxax 2sin A cos B=sin (A+B) +sin (A -B) 2sin AcosA=sin2A i = 7 J 2sin Bx cos xe 1p. 5, ly. =] Join ae + sin 2erde ~ 5 fsin 6rde tiv) fee a= [Peete ae (sin 2A =2 sin Acos Al cos 0s (w) [8242 gy sin x sin 2A = 2 sin A cos A] 4.gin x cosx cos 2x dx = feos 2x cos x dx in x = By [+ 20s Aces B = cos (A + B) + cos (A— B)] = 2f Loos (r+ x) +008 (2a = x)ldx INTEGRAL CALCULUS MADE EASY sin nas =f cos dx dx +2 f cos ede a2 +2einz +e =F sindr+2sinz+e. Example 25. Evaluate the following integrals : @) Join dx sin Bx dx (i) Joos 2x c0s 4s cos 6x de. Solution. (i) [sin 4x sin 8e de = 2 fasinarsinas dr (Multiply and divided by 2) = } flcos (8x42) ~c0s r+ Azide {v 2sin Asin B = cos (A —B) —cos (A + B)] = aliens co8 128) de = 3 feop dxdx—3 feos 12xdx 1 sin de sin 12x40. Gi feos 200s 4x cos 6x.dx = 3 J(2e0s 4x.cos 22) .cos 6x dx [Multiply and divided by 2] = 3 Jitcos (4x + 2x) +008 (dx - 2e)]. 008 6 de [v2 cos A cos B = cos (A + B) + cos (A-B)] = } J eos 6x +-c08 20). cos Gx ax 1 2 = 5 J eos? 6x +e0s 6x 008 20) dr =] feos? 6x + 20s Gx cos 28) de [Multiply and divided by 2 again] = 4 ft. + cos 12r)+ cos (Gx +2x)+ cos(6x— 2x) de [+ cos 2A =2cos*A-1=9 2 cos? A= 1+ 00s 2A) 1 =i fi-de+ feostze dz + [eos 8edx + feos 4x ax] = d[>+ SR Hae SEE 4 “a 12 8 4 Example 26. Evaluate the followi of Teme Wo | ae integrals : FUNDAMENTAL INTEGRATION FORMULAE 45 (itt) J sin fT eos Be dx Gio) [sin mx cos nx de: cosx i) | ————ae Solution. (i) J T+cosx =f Meta har [Add and subtract 1 to the numerator] 1+ 608. =f (- 1 Jas T+ cos x =f tar- ds Treosx 2A ‘(1400s 2A) =2 cos” A s+ L+e08 A =2e08? 5 x _ ze aye 2 =f lax Ff sec? Sadr ex-F}—2| 40 2 tan = x-tan 5+ wf 1 Le ete2A=2 ene" A 1+ cos x > 14608 A = 208? > +e Git) J sin x T= ea Be dx =f sin xJ2ain? x dy [= 1-cos 2A = 2 sin? A] = Bf sinxsinxdx= V8 sin? x dx . 2 2 sin? xde [Multiply and divided by 2] og] d-coanae = a L.de- J cos2x dz] . («Mase 2 INTEGRAL CALCULUS MADE EASY Gv) J sin mecos nx de = iJ 2sin ms cos nx dx [Multiply and divided by 2] Ff sin on + ne # sin (m= mel de [+ ein A cos B= sin (A +B) + sin (A~B)] 1p, 1p = 3! sinim sme de+ of sin (m— nx de 4 cits) 4 Saree 2a) saee tah meee |e (m+n) 2 (m-n) (m+n) t (m-n) Example 27, Evaluate the following integrals : of Tost £4 9sin® Too eto sin ® ae feet [t= cos 2 dx 3sin® x cos” x 3[eete= seman alc ee | te. iy J sin“ (cos #) de wf - +aing % x 49ein? Solution. (i) { 18 2 +9sin_ = Bein? x cos* x Ay (ee tenet x Osin? x “3! (sin? eos? x * an! con? x 4 Tess, sine) sing.sins * cos xcos x 7 9 =f cence xeot rde +2) soc xtanxdx Bp cose x + B sec +c. i) f 0a T= cos te ae = J cos xysin® » ds [> (cos 2A) = 2sin? Al = V2 sin xcosx de = By 2 sin xcoaxde [Multiply and divided by 2) =a] sn2eae [+ 2sin A cos A= sin 2A] FUNDAMENTAL INTEGRATION FORMULAE 47 at OE yen J. cos 2 +e 2° 2 v2 : Gi J sin"! (cos x) dr =f sin [sin(Z-a]e [ sin($-0)-cose] =§ (F-<]e = Sf def nae sin ww | Tein =f (eget) ae [Add and subtract 1 to the numerator} 1+sin x ={(Hae- 1 Jatx = J tae] 1 ay l+sinx I+sinx l+sinx ef rae -f taxa {On rationalization} Tesins ~ I-sinx =f tae- tasinx 1-sin? x l-sinx =f tde~ ds Jide [ners L sine ‘sin? A +cos A = fiae-f/—-- dx J Iles =) ey =f tae-J sect ede + [sees tan xdr sx- tans +seexte. Example 28, Evaluate the following integrals : Gi | 20082 J sin 2ec0s avd wea? ain? x nt ii) cos 2x+ 2sin © ae ti fan io. Eat lle cos x T+ 0058 (©) f tan” (see x-+ tan x) de. Solution. () {sin 2xcos Sx de [Multiply and divided by 2) 48 INTEGRAL GALCULUS MADE EASY = Pf fin e+ 2e)—sin Ge — 20] de (eo 2cos Asin B = sin (A+ B)—sin (A-B)) = 4) Gindx-sin sx) de = 3{f sinss arf sinxdx] “2 “2 = scot] sen con se deosete. 10 *2 i f sae cos® x sin® x [+ cos 2A = cos? A ~ sin? A] -2/(aats. sin? x a | cos? xsin? x cos! x sin? x vile cheer fm =-2cotx-2tanzte cf (emeszein’s x cos? x ~ 2 feotx+ tana] +c. [cos 2A = 1-2 sin? A} tiv) J sin® [=n rSae =J as n-[ =e «J sin® [tan (fan 6) a8 = [ sin® [tan tan ]a0 = [ sin’ ode eat cnt ses fe 1 1 @ 1/(sin 26 =3) 1d-5) cos 20.48 = 5 ( ; Jee 1=cos 2A=2sin? A 14008 2A =2e0s? A 1,1. = 99-4 sin 2040. FUNDAMENTAL INTEGRATION FORMULAE: 49 wf tan”! (seca + tan x) di “ sin =f tan2{_1_, Sine osx” cos x 0s 2A-= 00s? Asin? A = cos A= cos? 4 _ gin? A 2 2 sin 2A=2sin Acos A A A A > sin A= 2sin > coe = f tan “le [ Dividing numerator and denominator by cos 3 warfin(ZoS)ae [ml a) lde+ Ff ede 10. 12, M. 1B. 16, 17. 1. L INTEGRAL CALCULUS MADE EASY EXERCISE FOR PRACTICE Integrate the following functions w:rt. x Q. (1-7): (x Gxt aii) ot apy 2 ave (2 tin (x42) ie we(1- 3) (p(x = 1) (at + 2) tin [3p +00! c+] x (gest i Je +1 (+x) Yl-a wat Evaluate the following integrals Q. (8-20) : j dx %(24fe-nar j 1 7 uf 1+2x? faced + fae 8 Faye Jirreras 18, | @-59G+ 20 (1-Hde @ Jeesin s+ sds Ui f sin 2 + con a} ae lip [esee? x + cose xi de (id J dsin x ~ 2008 x4 soc? x ~B eonee? x) de (i f Ji cos Be de wo f T+ cos 2x de J tan scot 29? de fee 1+cos 3x 20. cos? me de Answers A 1 nt @gwre w- ar +e aid Be +e 2 Mee i) 5 84 del 2617-6 . 2 @ Zte Gi) log ix +e FUNDAMENTAL INTEGRATION FORMULAE & x+3loglr-3l +e 1 + yee _ (ae — 2982 10. 7g [eae + 20M — (ay - BP] +e 12, eee rte 2 2 3 4. -Reoers +e 15. (i)tanx—eotx+e 16. (i)- JZ cosxte 17. dtanx-Deotx-25x+e 19, -2cosxte 51 w-4 stan re be ¢0 x 2 GNF +e atx? Gr + 18. 6-7 aye Gia - 5 cos 2xe+e 2 Gi)-3 cos x-2sinx +4 tanx+5cotx+e Gi) JZ sinx +c 2 Integration by Substitution—I 2.4 INTRODUCTION In the previous chapter, we have considered the problems on integration of functions in ‘standard forms and the problems involving combinations of these functions. Often, we come across functions which appear to be quite simple, but do not fit in the category of standard forms. So, we need to develop a method of integration for those functions which, otherwise, cannot be evaluated directly by the use of standard formulae. These integrals can be converted to standard form by the substitution of a new variable. These substitutions reduce the compli- cated integrals to the standard forms which can be easily integrated. ‘The method of evaluating an integral by reducing it to standard form by some suitable substitution is called integration by substitution’. 2.2 CHANGE OF INDEPENDENT VARIABLE ‘Theorem 1. Ifthe independent variable ‘in the integral [ fiz) dx be changed to by putting x = (2) ; where (2) possesses continuous derivative (2), then J Pardes J fete ye). ae. Proof. Let us consider that 1 =| fadde, ai then, rhe) AD Assume that ; x= $e) ouch that dx _ gt Ee (2) «We may write that dl_dl dz ; ; al at de [Using equations (1) and (2)] dz dx dz = fix) #1) = flgiz)) ez) 8) Le x = 9@)] =a function of z. 52 INTEGRATION BY SUBSTITUTION—I Integrating both sides of equation (3) w.r.t. 2 ; we have I= J Age) 2). de = J feode= J fiee)).g'e). de. Remark 1. To evaluate J fiz) dx by the substitution x = ¢(2). Working Rule : (i) In the integrand, put x = (2) and dx = $2), dz. (Gi) Evaluate the resulting integral in z. (ii) Express the result in terms of x from x = ¢(2). ‘2. There are no hard and fast rules for making suitable substitutions. Experience is the Remark: best guide in this matter and one learns only through practice. However, some useful suggestions for standard substitutions are given below : 2 If integrand is of the form flax + 5), put ax +b =z t 12] pode dlax +b) = dz = (@+0)de=de = aot, de, (G) Wthe intogrand is of the form ="). fle"), put x" =2 det = dz > nx) de = de = deed de n (iéi) If integrand is of the form [Ax)P . f(x), put fix) = difix)\ = dz = f(x) dx = dz. Gv) If integrand is of the form o, put fix) =z = dlfixil = de = Fe) dx = dz (e) integrand imverese, pote = d(e*) = dz = ef. drm dz. 2.3 TWO IMPORTANT FORMS OF INTEGRALS Theorem 2, The integral of a fraction whose numerator is the derivative of the denominator is log |\(denominator) |. £@ a. dog Viz) +6. ie, te - Proof. Let us put = fix)=z > d [fix = dz > ('@) da =dz fie) gf ~ roel = elogs = log if INTEGRAL CALCULUS MADE EASY Theorem 3. The integral of the product of a function raised to an index other than (~ 1) and its derivative is obtained by increasing the index of the function by 1 and dividing by the new index. ie, J itor. pe) dee CO ye ae, Proof. Put fil=z = afta) = dz = fe) de = dz = J por pede fa et “(al lv ne-11 _ rear n+l 2.4 SOME STANDARD INTEGRALS Prove that : 1. f tana de = log | seex | +e 2, corxdz=log | sinx | +e 8. fsccxdr=log | secrstanri+e 4, f coseexde= log | coseex-cotx | +e = log tan (E+ 3) +e = log tan +e Proof. 3. Let I= J tanxdr = r= f 2 AD osx Putting cost=2 = ~sinzde =dz = d= sine We have Ts 22 (54) .a= fda cos ‘(sin == log Iz +¢=—log loosxl +¢ Le z= cosa = log (cos x41 +e Ls mlog m = log m"l = log Isec xl +c. 2B. Let ts J cotxdx | = =f ae sine Putting singaz = ewosxdeade > de=—— ae | cosx INTEGRATION BY SUBSTITUTION 55 We have 127% “cosx = log Izl +e = log Isin xl +c, (: 2=sinz} 3. Let I= f seoxde j seateeztins) sec x (Sec x + tan x) de (seex + tana) [Multiply and divided by (sec x + tan 2)] Putting (see x + tanx)=2 = (sex tan x + sec! x) de = dz = x= ——_1___. (see x tan x + sec® x) sec x(sec x + tan x) 1 & (seex+tanx) "see x(sec x+tan x) =f dig z = log lzl +6 slog isecr+tanal+e Ak) [> 2= (seca + tama)) ‘We have Is Also, we may write that 1+sinx “ cosx (cos? +sin? 5+ 28in § cos) “ eost A tein? A=1 Le 2 2) sin 2A = 2sin A cos A 2% _ sing = 2 ‘in? cos? J sin? cos 2A = cos” A—sin? A a (cos +sin 3) {cos # +sin (cos =~ sin 2 2 2 2 1+ tan = = 2 [Dividing numerator and denominator by co 1-tan 2 seca + tana = tan (5+ 3) +2) Putting this value of (see x + tan x) in (1), we get = og tan 56 INTEGRAL CALCULUS MADE EASY 4. Let 1s f coseca de. cosec x(cosec x—cotx) 4, (cose x — cot x) (Multiply and divided by (cosec x - cot x)] = Is Putting cose x—cotx =z => (=cosec x cot x + cose? x) dx = dz = —1____, cosec x (cosec x — cot x) cosec x (cosec x — cot x) 1 Wehave T= | Gosee scuba)” cosee a ensec x= cota) ° 1 efi z log Izl 40 = log looser a - cot #1 + wl) [v2 coges x ~ cot x] Also, we may write that 1 cosx 1-cosx cosec x — cot x= ——— SE sins sinx sing + 1-cos2A =2sin? A in? = __ Pain = 1-cos A =2sin? ” gue = 2 2sin 3 cos * and sin 2A=2sin A.cosA n cosee x — cot x = tan 5 AQ) Putting this value in (1), we get 1s | cosee x dx =log 1 ‘ Remark, Let Te J cove rds f oo de (+ sin 2A = 2 sin A cos A] ain ‘limtar* 2 2 [Divising numerator and denominator by cos 3| 1 x sec? = 5 sect = “Joe 2 ude 2 «ug tn 2) tnt wg [an 3) += tn INTEGRATION BY SUBSTITUTION 57 25 PORTANT NOTE ‘The students are advised to remember the following results : @ | sinxdx=~cosx+e (i) | cosxde=sinz +e iit) J tan x dx = log Isec xl +¢ (iv) | cot eds = log Isin x! +6 (v) J seex dy = log tsee + tan 21 += og | ta E+ 3) +e (wi) J cosee x de = log lensee ~eotx1 +€= og | tan Z te. SOME SOLVED EXAMPLES | Example! L ial uate the following integrals : On ee peas de Wi J Gx+5y7. de wit [aga (iv) J 2esin a2). de og )* »{ —1_ (wy 5 Mee de wf arskga 6x-8 I= Solution. (i) Let = = ~Bx45° Putting 3x? -8r+5=72 = (6r-8)dr=dz We have teft.a z = log 121 +e log 132? ar 451 +6. fs 2230? 45) Note. If the numerator is the derivative of the denominator or a constant multiple of the deriva- tive of the denominator, put the denominator equal to z. i) Let 1s J @e+5t dx Putting Be+5=2 = 3.dred: > dr= 1 1 . -( iet.aetfz «We have taf 5G den Ff ode ae 1s agp tee gy atte (ae+5)8 oP te 24 . Iv z=3x45]) INTEGRAL CALCULUS MADE EASY iii) Let Putting = We have fy 226-20] (iv) Let Is J aesin@? + 1).dx Putting Belaz > tedead 2 We have Ix f sine de =-cos2+e = cos t+ I +e, zex41] (w) Let If (og =)” ay x Putting logeaz = 2 adx=de a We have Is f #.dez% +e = 2 fog i +e. Ee z= logs) . 1 1 (oi) Let 1-f yralogs I saves * Putting (+ loga)=z = t adeede We have 1=f tia 2 = log Iz +e = log 11+ logs! +e. [fy z=1+logx) Example 2. Evaluate the following integrals : 2 ( | cos +9). de Gi f B® ae * ae emtan'x 2 ain J Tag de oy f =e INTEGRATION BY SUBSTITUTION—I 59 Seosec” x T+cotx | wf ro eel (wii) J aye Solution. (i) Let I= f cos (2x43). dr Putting Q+dar 1 = Rideade = dx=5.dz 1 We have I= J 5 cosz.de 1 1. a5] cosz.dew 5 sine +e = J sine +3) +0. [jy z= 2043] (i) Let Is] sec? (log 2) de ¥ Putting logea2 = 2 .deede 2. We have Ix f secte.de=tane+e = tan (log.x) +e, fy z= logs] (iii) Let Putting We have [+ 2=tan 7x] 2 Gey Let see? x ” B+tanz Putting tanx=z == sectadx =dz We have 1s [ ct -de=log 15421 40 Bee slog 15 +tan x! +c, 1 1 Let Is drs dx " jyd ee) rt e a 1-f See INTEGRAL CALCULUS MADE EASY Now, putting 1+etez = efdeade We have 1a [2 de=tog iz! +e z slog 1+ e%l +c. te zelse) © cosee? (vi) Let 1-/—= 1+cotx Putting l+cotxr=z = ~cosec*xdx=dz = cosec?xdx =~ 2. We have t2f-S.ae-9f tae z 3 == 9 og lz! +0 == 9 og 11 + cotxl +c. [eo z= 1+c0ta) (vit) Let S41 ay [Dividing numerator and denominator by e**]. git get = Is dx= f oa ae Putting e*®@e*% = on). dx=dz => get tem) deadz = (824+ e%2). de =2de 2 1 z We have te f2aen2f 2a =2log lal +6 = 2 log le -e*71 +c. te zane? e747] Example 3. Evaluate the following integrals i-tanx a i ©] ae | Fae i a ; i id [ ae fae? de CN ae (©) | far¥B de (vi) f (4e+2)( Jet ved) de, 1_sin - d-tan x Solution. Let t= f Trae de ‘ Ts dt Putting cos x+sinx =z INTEGRATION BY SUBSTITUTION—1 61 = (-sing+4cosx).dv=dz = (cos x—sin x) dv =dz We have 1s [4a = log Iz| +0 = log eos x + sin xl +e. (sz =cosx + in x] . 1 ii) Let Is edz @ Iran Putting tae 9 0+ 32" dreds 1 1 = Be utd > Fedendds We have tea [tae z =2log Iz| +0 . = Blog 114 Jel +e. [se z2=14 Jz) (iii) Let Ts J 4x Ja-2" des f 5-2? . andr Putting 3-s8=2 = (0-2)dx=dz = ~2ndr=dz = Qedx=—dz = Aede=-2%de We have Te fF 202) 2-2 f ode am aan 4 =-Fatre =- 46-27 +6, bs 22@-2)] , ft . 1 (iv) Let =| eae *=) ae dz Putting Maz = ret = dra6d.de We have Te] 1 65. de 2+ Ut-241 242 242? 5 3 = 2 dew6 { 2d ‘ia [a = 1 -2?-2 =6 2 —-—_|.dz + + S[e z+) Al 62 INTEGRAL CALCULUS MADE EASY (w) Let Putting We have (vi) Let = Putting We have off # de~[z.de+f t.de-[f oh | =6[ 5S se—togizoat] +e 3. get = ee Glog lz +11 +c = 23922 4 Ge Glog Iz +11 +e = 262) 3 28? + Go — Glog Ix! + 11 +c = Qe? _ axl 4 x6 _ 6 log 1x48 4 Ll +e. Is | farvd . de 1s f (ax +5)? de axtbaz = a.de+0ade zox)8) a.dende > dew>.de refer Lae feta fs zsaxr+d). T= f ae+2) Ix? 441. de Ts f2ae+ 0. jet eeel dx exelaz > Qr+1+Odrade > (r+ Ddxede 122 ie .dr=2f 22 de 32 2 +en2|“g-|+e =2 oN I 2*? 2 ot = +e a we tat DM te, zeX4xt i) calm cole INTEGRATION BY SUBSTITUTION—1 63 Example 4, Fvaluate the following : » x - I . ol ae i | Saogge Ot 2>0- ay ( tye” it 4 an | TE” ae (iv) J sect x. tan xdx. 2 Solution. (i) Let =| aa +3r Putting 24B8e2 = O+etdeads = atdred de We have lv 2224 3x5] Gi) Let logs de 3x >0. (log a) Putting loge-2 > fde=de x. We have Te f Lide= fe? de # 1 * = +e==—4e =-p+1 l-p 1 = OE Ae be z= log x] i-p iii) Let t= [2 ae le Putting l4vz 22 = lear = (ordev*) draae ae den de h de = 202 = aged = Je ase . We have La] 2 ddew2 | or de art =2i—4e n+l 64 INTEGRAL CALCULUS MADE EASY a = eet b- 22142) net iv) Let 1s J sect x tan xdx. = 1s J sect. sec tan x. de Putting secr=2 => (seextans).de=dz ‘ We have I= J.de=Z re see! x . +e, z= see] Example 5. Evaluate the following : 3 “lea «wf fan x * Tog ieee x-sinz «io J log log Mog} * oles oweains de toy | in vlan x Sin x08” chen Solution. (i) Let = fee : eel = Putting ele: > eele-1) = (40).deadr => deed: We have 1 2 az nz=log Izl +c z | = (e+ 1)=log let + LI +0, ty eseel] | i) Let =f me, log sec x | Putting log sec x =z = 2. ieccxtans).deade + tanede=de see x | We have 1s J 2 .de=tog tzt +0 z = log Nog see x1 +e. [se 2 =log see x] INTEGRATION BY SUBSTITUTION—1 65 a _ i ii) Let 1=J shysintgal Putting log (log x) = 2 to) geade = dx de log xx slog x 1 We have tf Cog (log =)}.(x log x) ~ =f J de wtog tz +c z = log Hog (log x)1 +e. [2 =log Cog x)] tote taf SECA Putting cos x +sin x= 2 = (-sinx+cosx)de=de = (cosx-sinx)de = de We have 12 J 2 -dentog ict +e wing ems ins Ls z2cosx+sinxl (v) Let Is jm . Bin r eos = [Multiply and divide the denominator by cos x] ytan =f eee J SEE cette {S82 com « tan cos. =f 1 sectxdx = J (tam x). see?x ds Jean = Putting tanx=2 => secixdy=dz Ht 4 We have 1s J 2% de=|=7—|+0 -341 2 =2224e=2VF 40 [yp estan x] (wi) Let Putting e+retaz = (e-ede=dz We have 12 f 2 .de=t0g lelae z = log let*+e*1l +e. (te zee +e*) (vii) Let Is J sint x cos x de Putting sinz=z = cosxdr=dz INTEGRAL CALCULUS MADE EASY ‘ + We have Inf tide ti ve sin x] 1 = qsintxee. t 4 si x (vidi) Let t= f Gap Putting G-Oe2 3 Bel-2 > atdes-dz = xtdr=-$de ® x ose Gy? Jean a-2? (- <. We have =f fy x=1-2] Ly (ab) = a? +b? - 205) i 3 2 “2 -ja- 2 —Fa- ayy 2a Dg [ zel—xl] | Example 6, Evaluate the hollowing integra : 1 _. agent | © | aogz-® wis at as. © Solution. ict =f Togs de Putting logx=2 = 2 dreds INTEGRATION BY SUBSTITUTION—1 1 We have T= J 5 de =tog lel +e = log Hog x1 +. lv z= logs] (ii) Let 12] ie . Putting Beetar = +e deed: = edrade . We have t= J 2 ae=tog tet +e = log IS tel +c. z=8 +e] (ii) Let 12fi "2 & [Multiply and divided by e] wre’ ely geh Lge? = red e@ttte) gids tettre e xi +e* e xi +e™ Putting Stetaz = (ext +e). dredz We have tof 2 deat ig tat te etz e =f hog let tet te. ty z=xt +e] Example 7, Kvaluate the following : (J oe ats de (ii) [ tan x sec? x. de ar _ gk «Se soe? a { one ip) [ tant fx see? Jz oe ae ode cio f = cw) J S cos (0) | Join Be -cos 2x. de (oi) | Se - de Solution. (Let T= J Per (t a 11 dew fet” 1 de x1 3 af os ghes! a Putting eler + (eds 0ideode = Addon ds + dent de :. We have ee ee deat fiia a ale 1 1 = flog Izt #c= flog Ix4+ 11 + Gi) Let Te J tan? sect x de Putting tanx=z = sectzdz=de 4 We have Is J tae se INTEGRAL GALCULUS MADE EASY =F tants +e. ty z= tans] ai et te (ii) Let r/o Putting =e tear = (2-262) deeds = Weeder de = Ce dea hee We have Ta} f Pde 5 log te +e =F log te eel 40, fe zee pe iv) Let 1=f tant Jz sect Ve gp Ie Putting tan (Fez sect ie LG) we i = se? fet Ldred = “Mere ROE dz 2 . 1. We have Ina ft.dea2= re 5 =2 eG 4 z=tan Jz] = Ztan® JF +6. (v) Let Ts J Yein Be «005 2x. ce Putting sin2x=z = 2cos2edxr=dz = cosI2xrdx= 2 We have Ted fe dee 3] de Ay cig?) ag aire 2 1. . = 5 in Be? +, ty z=sin 2x] . cos x (wi) Let 1: | har Putting Ltsinz=z =» (O+coss)deade = cosxdesde «We have 1s f fede fae INTEGRATION BY SUBSTITUTION—1 69 stp tee dette “ie =2(L+sinz)* +c (y z=1+ sin x) =2 ivan +e. Example 8, Evaluate : » [ @+32) a wf Gay wi J a 3 , sec (iii) | cae 4-2) ae wwf lg tecrsmmy' 2 — Solution. (i)Let I= J 5* Putting ($=2x)=z => -2r=z-3 > 2. We have = Bf dace 8 fade 13 3 2-7 bonita ees ~B tog 3-201 + 3 G20) +0. by e292] (di) Let I- i fas = 1=f——te we (Multiply and divided by 2") xe yx? -1 Putting slag = Qtdeede + xtdend de 1 2 We have 3) aa INTEGRAL CALCULUS MADE EASY co wteocte+e wpoectstec, be Git) Let T= [dea fet rel de Putting «o atxtlaz = (2e+1+0)de=de = Qr4Vdeade = 2x41) de= dd = (de +2) de = 2de. We have I= Je. 2de=2 f 2a 1 da 7 =2 i tent ee Aut 5 2 4 apt tae Pee, te zeatesell , sec x (io) Lat 1") jogGecz+tany Putting log (sec x + tan x) =z —1__ see x (see x + tan a) > ery (eer tans acc! s)demds —» MERMeest ten = see x dx = da We have t= J 2 den tog tai +e log Hog (see x+tan sil +c. fr zm logiaee x + tan x)] Example 9, Evaluate the following integrals : pon af 2 @ de olin ole ag) [ 2o0sx—S ain x . 1 it) | Gece x 4 sin x wa J woe? xd —tan a) sine WOR 4 sy f Lt eos © | qa * wo f fizz (8 | TH coas oto Betotion() Lett = {Fy ode 1 Putting sintze2 = ode = dz ie We have Infet.deceree sett ae, [y 2=sintx] 71 4 (a*)2a" toga | Cy 4 (2? = 2% = (28)? = 24] a aintete fe 2224 (iii) Let Is +f Bei, dz Geosz+4sinx = 1-1 ' 2 Putting —-« Seosz+2sinxez = (-3sinx+2cosx)dendz = (2008 x-8 sin x) de ade :. We have tel fl dee} og tei se Ql2 2 1 = 5 log IS osx +2sinal +c. f+ 2=8cosx42sin 2] . . 1 see? x (iv) Let 1-| Gyacmen oer: Putting (l-tanz)=z =» -so@xdred: = sec’zdr=-dz 1 t e-| a se 2 =- 2 We have 12-] @.d=-f tae =-[-3]+e-J +e z z -«—1, (» z=1-tanx] “(tas * 2 * (v) Let 1-f aves? : Putting (l4cosz)=2 = —sinxdredz = sinxdx =-dz. 1 We have =-[ =. de=-] 2? dz Jz J ts z=1+ cos x) INTEGRAL CALCULUS MADE EASY af diez 4 (flex ite sonalizatic (vi) Let Is jfe. axe (= fe) a [On rationalization) =f tx ee Ta* S5 > dx -D) vae[gtze Putting lastez = -20)deeds = -2edeude 2 rdea—Jde Putting this value in equation (1), we get Tesintx+(-fi-2*)} +0 ssinx- 1-2? +0, (wid) Let Ts f HES ae cost 2cos" = ue =2cos? “J = de : 1+ 08 2A=2.os: A sin 1-cos 2A=2sin* A = | (coon ) as Ls cosec? A- cot? A = 1] cot (x / 2) Ve 7zte INTEGRATION BY SUBSTITUTION—1 73 Example 10. Evaluate the following integrals : fa 1=cot x of peer «oJ there: Git f je wh Gv) | Ctr sabe an 12) dy tanx a | Shar ze wo f Trumt Solution. (i) Let I= [ Fe 2 =f 4 ay = -de Dividing numerstar and denominator by e Putting bet+e=2 3 -betdead: 3 etdy=-=.de d. a Webave Te f £.(-f:)=-$f 2dee— tog tel +e; ~ flog Ibe* acl +o, fe zebet+cl 18x a 1-cotx sinx Gi) Let Ie irae * | oe sing - sin x cos x sin x + cos Putting sinz+cosx=2 => (cosx—sinx)dr=dz = —-~(sins—eosa)dx=dz = (sinx—cosa)de=—dz We have I J 2. ce=-tog 121 +0 = log Isinx +008 x1 +0. be z=sinx +c08.x] iii) Let te f ttleet ae 34a logs Putting (B+xlogx=2 = [ove 2etoga.t}de=de * = (1 + log x) dx =dz 1 = We have T= J F-de = tog tel +6 = log | 34x loge | +e. 2=34% logs] (iv) Let Te f antonio | de +a? Putting tmtee: = —by deeds 74 INTEGRAL CALCULUS MADE EASY 2 Wehave I= singe.de=- 3% _i-tan?A 1+ tan? A : tan rez x=tanz {v) Let * tf acos? x +0 sin’ x Putting a costx + bsintr=z => (2a cos x (- sin x) + 2b sin x cos x) de = de = (-2asinx cos. x+ 2b sin x cos x) de = de = 2b = a) sin x cos x dz = dz = sin x cosxdx = 1 1 *~ Weave Ts Se ae-w ae areal: slog lal +e log la costx + bain?el+e. [, 2=c costx+bein?z) “Bo a) “Te a) (wi) Let tf ieee] Lf _2eos . i =3) meh # [Multiply and divided by 2) (-sinz-cosx) 4 (cos x— sin x) [Add and subtract sin x to the numerator] aff tar 3) SRE Bx ie cos x—sin sin x cos x -ia) Gea @re a Putting coaz—sinx=2 => (—sin x-cogx)dz =dz ~ We have leo-3 daere INTEGRATION BY SUBSTITUTION 75 = 5-3 log zl +0 = 5-2 tog teos x sin sl +6. Le zecosz—sine] Example 11, Evaluate the following integrals : to j Mes Dees toes ay (Gi) J cosee x tog (oosee x ~ cat 2}. dx titi f Zpsin(2). ae (iv) J Je*=1 dx. 5 Solution. (i) Let Te j Met Det legs ds = =f 4(4). (+ log x)? dx = af(red 2) stops? de Putting aehgeez = (2+2)ared We have 1-4] 8 .de 4 z 4. +e=2tec 4 = (x + log x)* +0, {rz =2+log x] (ii) Let Ts J case x log (cosce x— ot x) . dx Putting log (cosec x — cot x) =z 1 => (eoucxsote) + © cases x cot x + cose? x) dx = dz cosee x (cosee x — cot 2) ecstasy" = cosee x de = dz We have .dz= zee = 3 log (cose — cot 2)F + [vz slog (cosec x — cot x)] 78 INTEGRAL CALCULUS MADE EASY te 2elee) {Add and subtract 1 to the numerator] de] «212 -tar zl+e =2 [ford tan (e=A] +6 Example 12. Evaluate the following integrals : ‘ 2x ) sin ode w 7 Sa a? sin? x+b" cos" x” 2-(e=1] wl Ee 5 sin (tan"! x4 Solution. (Let T= f 824) ae Lex 1 Putting tartar 3 Ter ana ye de ade dead: > Ve’ “a GF We have 1=3f sinz. de =} Ceos2)+e cima [fy z=tan-l xf] in 2x it) Let l=) = a: J taint ebb costs “2 Putting a? sin? x + 6? cos? x =2 => (2a? sin x. cos + ~ 26% cos x . sin 2) de = de = 2 sin x eos x(a? —B2), de = dz = sin Be de = ys, «de wa: INTEGRATION BY SUBSTITUTION—1 7 We have en =.dz= 1 72 wy jzlee log 1a? sin? x + 8? cos* x! +c. ts za? sin? x + 6? cos? x] ii) Lot Ie iii) js. zevest Putting vetlsz = etle2® = xe2-1 = desk 2 2 We have ref APA ae = 142) 2 =2 & Bt ae 2+ _ - 34, 2 =2[ 272? a 2e z+ P41 e+ = = =2 flies 22 | Sei = 1-- . af {s 2 5) de 22 oa =2f2.de+2f t.ae-2f =. -2f 5: 2 2. 422 -2log let4 11 -2tantese a 28 42— Blog 1274+ 11-2tanle +e (v= Je+D =Q@zt) +2 Vx+1—-2log lx+21-2tant Jx+1t+e ; 1 (iv) Let ‘Ge (vz - Jz+1) (On rationalization} Sees (G-ar) * of PAF) gp BET, (x+D =-[ Weace] fevt des xt des f +0 de 78 INTEGRAL CALCULUS MADE EASY 1 1 a det xt (2+D? pel fel *e a Fats Fat ee, Example 18, Evaluate the following integrals ; sinx | gate Sin@e—a)” a I 00 | ec aicee 28 a I - sin 2x wo | Seaamecw of Free desa-beOn+ D5 sneZ T . i wf ae e wo f coe =a) coste—6) sin (x +a) wi) | ab sin (e+) * Solution. (Let I= { "= . sin (x-a) Putting x-a=z = xecta = desdz + Wehave I= S2Gte (+ sin (A + B)= sin Acos B + cos A sin B) sin 2 cos a + cos z sina sinz sin z cos a cos zsin a FES gee f SOR28ING gp sinz sinz ie =f cosa.de+ J sina cotz. dz =zcosa + sin a log Isinz| +¢ a) cosa + sin a log Iain —a)| +e. fy e=x-a] a 1 Tt Let =f — 1 ae ta g, 7) I aerate dej(a-b)eQn4+1) 5 ine % (Multiply and divided by cos (a - 8) 0s (a-b) = 1: Gach! mae-aeearw 1 cos I(x - 8) -Ce-a)] * cos (a-8) J sin —a)eos(x-2) ° ‘ (e—b)-(2-a) =-b+a=(a-b) ‘ ¢08(A—B) =cos A cos B+sin Asin B INTEGRATION BY SUBSTITUTIONS 79 _ 1 cus (s—a)eos(x-B)+sin(x—a)sin(x~B) ~ cos (ab) sin (x- a) cos (x - b) 1 Rae ) sin (x — a) sin (x | ds cos(a~ 6) 4 | sin(x~a)cos(x—b) sin (x —a)cas (x ~b) = og] foot a) +tan (x b)). de = ech [log Isin (x —a)| log eos (x —d)I] +e [: log m ~ log n = log 2) 1 sin (x -a) * cos(a-b) sin (x - b) Ie 1 woe I=) eee 1 . mera! si (Multiply and divided by sin ( - @).) - in [(x -a)-(e-B)] sin(b-a)? sin(x—a)sin(x—b) [: cnc Bade Acc om sain 1 sin (x-a) cos (x ~B)-cos(x-a)sin(x-8) * sin(b-a) sin (x— a) sin(x- 6) . _ 1 sin (x—a)cos(x—b) _ cos (x—a)sin (x —b) cde sin(@—a) J |sin(z-a)sin(x-6) sin (x—a)sin (x = GLa oot eb) def oot xa) ce] gy Hog Isin (2 ~B)1 — log Isin.e—adl] +e += dead f sec De de [+ sin 2A =2 sin A cos A] 1 sno sin (x - b) tg Se a) = sine ~f (iv) Lat tej soz. de sin 2x 2sin 2x cos 2x 3[ Ree aes) ‘te I = J log love 2r + tan 2s! +e. INTEGRAL CALCULUS MADE EASY 1 or 12) ea = J se cel de (Multiply and divided by /sin x] ¥! x.sin (x +a) 1 sin x “J wats Vance) de Putti sin (z+ a) =: tng Binz vin. x 900 (e4-c)— cow xein ix +a) dead: sin? x sin (A—B) = sin A cos B—cos A sin B] = ain [x ~(# +a] deh = sinC a) dv=dz sin? x sin? x = at den de sin? x = ja --|2 tee-ealtbe sina|_1,q sina | ste) z= fink+a) sinx (vi) Let 4 1] geacamaeah = taf sin@a—b)__ sin(a~5)4 cos (x a) cos (x— 5) * [Multiply and divided by sin (a - bY] - 2 sin ((x~ b)-(x-a)] sin(a@-0)4 cos(x—a)cos (x —b) * ty @=b)=(e-a)==-b4a2(a-5)) sin (x-B) cos * sin (a6) cos: (. sin (A-B) = sin A cos B - cos A sin B] INTEGRATION BY SUBSTITUTION—! at eos (x-b) cost: * aaeap! ante b)-tan (x-a)]. de si tan (xb) x= f tan (x~ a) dr] 1 = ina aby Flos Veos (x —B)! + log loos (x abl] +e : see-0) |, log m — log n= log ™ * sin (a ~B)| "8 | cos te —8) || * gm — Tog n= log sin (x +a) (vii) Let -f/See. Putting xtb=2 3 raz-b 3 deade «We have taf Smienbre sin z .dz =f sin z cos (a ~ b) + cas z sin (a —b) sinz [+ sin (A+ B) =sin A cos B+ cos Asin B] = J cos (a—5). de f cote sin (a—6).d2 = cos (ab) [ t.de+sin(a—2)f cot edz = 2 c08 (a —b) + sin (a—b) . log Isin 21 +¢ = (x +6) cos (a ~) + sin (a~ 6) log Isin (x + 6)L +c. ts zeG@+o)) Example 14. Evaiuate the following integrals : . p [ixsinx [eesin® Oi serres Solution. (Lett = f j2*8"4 dx 1-sinx eos? 5 + ain? 5 +2sin 5 cos 5 -f in? a) 30 2 ‘eos? A+sin? A=1 | sin 2A =2sin A cos A ii J i asin x +b coax cos? * + sin® Tate! 2 2 2 INTEGRAL CALCULUS MADE EASY (a+b) =a +b? +2ab (a~b)? =a" +b? -2ab 1 x Jeos2) de = dz 3°3) £2 (con +sin 2) ae =— ads 2 Wehave 1-J2.c2a)--2f 2 de=—2tog lel +0 ze gin= = z=0c08~—sin= sigan snd] + fe sound -an$] ap 1 aout t= | beer ® Putting a=ros@ and b=rsind On squaring and adding, we got a? + 6? =r? cos? 8 +r? sin? © =r? (cos? 0 + sin® 0) = ae bte? = r= faaee On dividing, we get & al? q zine => o= tn (2) We have 1 1 | cawosingereinbewe 1 1 =7) masm dr sin(A+ By sin A.cos B+ 008 A sin B] 1 a 2] cove e+ 0). de le) +e Hiog| tan (3+) +e ~ rin gge"(5) | INTEGRATION BY SUBSTITUTION _ 88 | 2.6 INTEGRALS OF THE FORM: | sin= x cos* x dx In order to evaluate the integrals of the form J sinm x cos* x de, we may use the following rules : (i) lfm, n € N and m is odd, then the substitution z = cos x is used. ii) Ifm, n.¢ N and nis odd, then the substitution z = sin xis used. (iii) If m, ne N and both m and n are odd, then either 2 = sin x or z = cos x is used. It is advisable to use z = sin x if m2 n and z = cos x if n 2 m. (iv) Ifm, n € N and m and n are even, then sin” x and cos" x are expressed in terms of sines and cosines of multiples of x. (v) If'm, n € Q and (m +n) is a negative even integer, then the substitution z = tan xis used. Let us try to understand the above mentioned Rules with the help of following solved examples : Example 16. Evaluate the following integrals : @ J sin’ x cost x de Gi | sin? x 008% x de itt) f sin’ x cos? x de (iv) J sin x cos? x ds (0) f sin? xeo6? x de (id J sin? x cost x dx. Solution. (i) Let t= J sin® x cost xd [Here m is odd] Putting z=cost > dz=-sinzdx = —dr=sinzds 4 Wehave I= f sintx. sin x. cost x de = J (ein? 2) cost x sin x dx (1 - cos? x)? . cos! x sin x dx Now by using substitution, we have =f 0-29". Cd), of A429 de sin? A + cos? A = 1) Ad-2P de J (a —b)? = a? + b? - 2ab) 9 9,7 - 8-28) dex -|2-+5 = Jee [z a Tite =i SabaZet se = eos’ x- 2 cos? x + 20087 ze ao Geos! x ons? x + Zoe! +c. (r 2=008 x] Gi Let Ts f sin? x coe? x de = J sin? x cos? x . cos x de Putting We have (iit) Let Putting « Wehave (iv) Let Putting + Wehave (v) Let INTEGRAL GALCULUS MADE EASY = J sin® x (1 - sin? 2) e0s x dx [> sin® A + cos" A= 1] zgesinx = dz=cosxdr. Y= f 80-2) dex f eee ae +e (ty 2=sinx] Ts f sinS.x ons? x de = J sin® x. cos? x eos x dx = J sin’ x (1 - sin? x) cos x dx [> sin? A +cos® A= 1) z=sinx = dz=cosxdx If 8a-2de= f 8-2 fs z=sinx) sin‘ x. cos? x . cos x dr = J sint x (1 ~sin? x) cos dx [ie sin? A + cos? A= I] zesinx = dz=scosxdx Le f A0-2de= f (et—-2%) de s wAgind x-2 sin? ts asi = Gain’ x Fsin” xe. f: z=sinx) T= J sin? x cost dr = [ (sin x 0s <)*. ds = J 4 (ain x cos x)? de [Multiply and divided by 4] =f @sinx cos 2? ae = Ff win20? ax [+ 2sinAcos A = sin 2A] INTEGRATION @Y SUBSTITUTIONS 85 vt cos 2A=1-2sin? A 2, _ 1-cos2A a2 f breed ay = sin? A= AREA 4 2 = > sin? 2a = 2 seats a1 1 _1__1sin4x = ff tde- 3 cosardr= 5x fate 11. = pin gg inde se, (wi) Let Is f sin? x cost x dx = | sin? x (cos? x} dx cos 2A = 1-2sin? A = sin a= 1720828 2 = [(HS2) E22) vd and cos 2A. =2008? A-1 2 2 2, _ Leeos 2A = cos’ Ae -{(=2") 1+ cos* 2x +2.c08 2x 2 4 ) a Le e+ bP =a? +6? + 2ab] = Ef cs 20) (1 + ons? Bx +2 008 20) dx = as (1 + 008? 2x #2 cos 2x — cos 2x — cos* 2e— 2 cos? Qe). dx = iJ (1 + 008 2x ~ cos? 2x ~ cos? 2x). de 1p l+cos 4x 8 2 =f idee d cos 2x de sf tide — Ef costed = Rf tte 3 f costed «de 2 F008? Bx , 608 Bx de — AY (1 sin? 22) con 2c de For last integral ; Putting sin 2x =z = 2estrdrad: = cos 2 die = & de. lsin2x 1 1 sin4x dz We have Asinde 1, Lsinde 16 j2)d2_ ae a ie ie a) nF 1 1 1. 1 1 = xt—sin 2x -— attigat A sin ts-4(2-=] +0 16 64 16 3 86 INTEGRAL CALCULUS MADE EASY sin@x x _sindx sin 2x, sin’ 2x le 2%, - +e z= sin 2x] 8 16 16 64 16 48 x _sindx | sin 2x “ie a * TS Example 16. Evaluate the following integrals : | sin? x cos xdx Gi) J cost x. eberin® de id) | sin’ x Joos dx. Solution, (i) Let I= [ sin? x cos! x de [Here n = 5 is odd] = J sin? xcost.x cos x dx = J sin? x. (cos*x)* cos x dx = J sin? 21 - sin? 2)? cos x ae Putting gesins = de=cosxde Wehave t= f 20-2. de =f Pate pehdes | t+ 28-22) d2 [: (@-bP =a + 6% 205] fv z=sinx)] (i) Let Ts f cos®x ebwrin= dx = J cos* x sin xdx [es ebef® = fax) Putting z=cosr = dz=-sinzds = -dz=sinxdz u Wehave I~ 2*.dz= . = 8 Fae, 6 (it) Let I= f sin® Joos de = J sin? x.sin x Jeosx de Putting z= cos = de=—sinzds = —dz=sinxds ss Wehave Is f (1~cos? x) Joosx . sin x de INTEGRATION BY SUBSTITUTION--1 87 = [ 1-22) Jz -de)=- | @-2) de J J sng = ge lr Byte | ste ptt ott 2 2 -[}27-2""] +6 onder 2 antes Example 17. Evaluate the following integrals : ; 1 yf et ate 219 x cos? x dx ® | Seer “he xcos?x (iti) J see” x cosecs# x dx ta § lan ade aa nee 1 de, oJ sin’ x cos? x Solution. (i) Let I J scat dx . sin®? xcos*? x * 28 = I= f sin? reos Bede Here (-2)+(-5} =-+ whichis a nogative even integer. 1 t=) arcane 1 =f ade sins e682 x cosh? e082 cod Sng con x con [ty Multiply and divide the denominator by eos 1} 1 Jarre ® see! -[ Ske] 2 gee? =f ee ae [+ sect A—tan? A= 1] tan’ Now, putting zetans = de=setsde 142? We have ref ig -\ar get | Se re d= f r%de+ fA de INTEGRAL CALCULUS MADE EASY 1/2 ao Rae tg mete 2 2 ty tan rte [+ 2=tanx} vtanz (if) Let T= f sin’® roost dy = [ sin®® x cos?x . cos x dr = | sin? «(1 ~ sin? x) 08 2 de [> sin? A + cos? A= 1) Putting zesinx = de=conxde Wehave =f 2°0-24dr= fe? -2% a wa = Ty te a * [+ 2=sinx] (iti) Let 1 =) aaa ede 3,5 sch Gs . Here tg 72 which is a positive even integer. 1 l= f eet laa 1 12 J = de Sn cos” x cos cos?" x {~ Multiply and divide the denominator by cos®* x], =J mec x des J SSE de Now, putting tanx=z = sec?xdr=dz Wehave 12 f 40 2 INTEGRATION BY SUBSTITUTION—I 89 -4 oe +e 2=tanx] (iv) Let 1. f - eral ytan x “J ane oa, cos x” [Multiply and divide the denominator by cos ] -[ few eae “Ibe: zetanx = de=sectede We have te feed fetes Now, putting (- z=tanx] (v) Let = J sin’ cos Fx de Here (—3) + (- 5) =— 8, which is a negative even integer. “ fae sin® x cos" x 1 lox ode .cos° x cos? x x [Multiply and divide the denominator by cos? x] 1 “latinas sec® x = sec® x. sec x tan? x tan? x 2 999 sec? (ian a2 ds [see A-tan?A = 1] nr 90 INTEGRAL CALCULUS MADE EASY Now, putting tanx=z = sec?xdx=dz 29 Wehave =f #2) ae z 2 4504 4.28 -f (he da" 452042 de [v (a+b)? al + Sa% + Bab? + 63] 2 wf [zeta S) cee [ete Ssaeee?] a sfetideesf hares fede) eae 2 2 2 z 32’ = 25 +Bloglet+ 2 Hy tSloglzt+ 5 wa yie tilogiels Gate Tet ve +Blogltan xl+ tan? e+ 4 tantree. [: z=tanx] =~ Stan? x Example 18. Evaluate the following integrals : (i) [ sect xde (iid f Jian «+ tan? x) dx. Solution. () Let I= f sechx de = f sect x. sects de = J (eect? sects de = | (1+ tan? a. sect de [: sec? A-tan? A = 1] Now, putting cz=tanx = dz =sec*xdr Wehave =f (+22. de =f setae ty afer fetider2f 2tde 5 3 wrt yg te 5 3 stan s+ Ztant x42 tants +6. [- z=tanz] Gi) Let Ye f Jana (1 + tan?) de = f flame . sects de [es sec? A - tan? A = 1) Putting zetanz = dz =sectxde Wehave If Jz de INTEGRATION BY SUBSTITUTION—1 ot 2 = fet ae # Stl aa -iatesg get ten 5 tans te [- zstanal Example 19, Evaluate the folloxing integrets + : W) J cost dx (ii) J sin’ de, Solution. (i) Let L= J cost x dx = J toosta? de = f(t) ae 2 1+cos 2A 2 605 2A =2cos A-1 => cos? A= (a+ bY =a? + 6? + 2ab} = FJ (1+ cost 22+ 2.08 28) de = } [1-23] 2eneas| wde al 1 =4 (1+3+2cos4x+ 200522). dx = 1) (G+ Jeoeaes zen 2s) de adla*e 3 1 1 =3) Lide+ 5 J cosdr. des 2 [cos 2ede = 34 Hinds , sin de “8 32 4 (i) Let T= f sin’ rade +c = | int? de segs) = 2 fa ecost ae 2e0 2) de Te (ab mas bt 0b] “aif (tae). 2eos 2x]. dx 1-cos2A 2 cos 2A = 1-2sin? A = sin? A= i (12 fs eon ae Beane). de “a 272 92 INTEGRAL CALCULUS MADE EASY 1p (3,1 = iJ ($+ Zeon 4x— 200 2x) ade =3f Lde+3f cos dx di ~5 f cos 2e de + al 4 Snot — sine te. 3(sinae) 3x 8 23 8 82 2.7 SOME SPECIAL INTEGRALS Theorem 1. Prove that of q 1 Gi) J eae Proof. (i) Let = f= 1 _ 2 “leper be @ 1 2a . aa! (@-nta+rn'™ (a — 6) (a + )) (Multiply and divided by 2a]+ = af (t+@-2) gy fr (ata)s(a—x)=at+e+a-x=2a] 2a? (a-x)(a+x) 1 1 1 $s (Zest [- log, m—log, n =log, 2] Gi) Let 1 -f—_i_. ss (a2 Bala l eaeee-* [s (@®= 6%) = (ab) (a +B) INTEGRATION BY SUBSTITUTION 93 =—| ———_.. Mult \d divided by 2c: Ps Resesrerey (Maiyply and dived by 22] al ta-G-a) a ~(e-a@)=xta-xta= “aad e-aerea “HL eted-te-aexta-sta=2a] el (A-sk)-« x-a x+a “alae alaa = gi toe teal - 2 og teat +e -a m +e + log, m—log.n=log, — ta a a +0, Sz ne (ii) Let ee Put x=atan@ = Zetne@ = =tan(Z). a a and xeatan@ => dx=asec!Od0 « a see 8 a _sec’® “ 1s] A aes 2 1). 8 (sec? A- tan? A= 1] =4] 1.d0=1 046 @ a =a [> e-3] zy 0 | pgp eon Soe wo | ahaa dente ea] +e (id J ope te mtog| x Yat vee vy J vdeo] geet = < 94 INTEGRAL CALCULUS MADE EASY Put reasin® 3 @=sint= = dy =a cos 0.d8 1 acos8 1+ [ +... acs 0a0 = | ——. a0 Ik - a? sin? 6 reece =f cos 6 - cos 0 4 eos? 8 cos @ =f i.dasore =sin?~ e=sin?= ssint ote sin! — wf opp een tint Ze, ro a (if) Let Put eeasecd => secd= = = ds =a sec 0 tan @d0 1 ‘ 1s] ey acco tan 8. do fa? sec? 0a a ee Ya? (eee? a- sec? A~tan? A = 1] yian -| see@ tan @ tan® = log Iaoc 0+ tan 01 +c, | =Tog | sec + Voce? @=1| +e, = log d6 .do= J see oda z x +Pge1 a a* +e, = log INTEGRATION BY SUBSTITUTION—I 1 = log | + =a" | — log lat +c, = log | 2+ Y= ? |+e J pts tea tog| x45 ea? | +e (Git) Let Put = 1 Ts [ sate ode faghs xeatan® = tano== di =a sec? 6 d0 1 2) . 2 cee? 0 0 I rrteate asec” 0 = - dé I Biers : -[ 2.20 You 8 -J wee apf sec 0d8 sec@ = log see @ + tan 61 + ¢, ? = log | J+ tan’ @+tano| +e, jog | fa? +3? +2] —og tal +e, alog|x+ (a? +x? | + J Srey anne oa" | 46, (iv) Let Put tof ahora x x xsasec@ = secOsT = O=sectT 95 [e sec? A-tan? A = 1) [se =ey—log lal] INTEGRAL CALCULUS MADE EASY 96 = dz =a sec 8 tan 0.0 1 ” I= -@ see @ tan 6 do | oe | asec 8tan 6 asec@ ya" (sec? 8-1) — Ls sec?A~tan? A= 1) *! aytan? 6 ° . Jr.co=2.046 a O=sec? = a L Lott “ dra see +c, Some useful substitutions are here given below which provides a great help to the reader for evaluating special integrals : Expression Substitution O@+2) #=atan Gora cot 6 (i) @-2) = asin 0 oF aos 0 Git) G - a?) =a a0c 0 or a cosee @ Gv) j= = a.cos 20 lave (w) je x=acos 20 a-2 wi) x= a.cos?6 + Bsin?® i) 22% = a 2 win (=F x= cost6 + Bsint 0 Witt) (2-9e-P x= acos?@ +B sin? 0 la? — 2? oe ® 2a cos". Example 20. Evaluate the following integrals : - i olga i Gin | See 1 Sear ‘ INTEGRATION BY SUBSTITUTION 97 1 Solution. (i) Let t= J <5 Gi) Let [> using J x-V7/3 x4J7/3 log, +o= 3x-V7 Br i7 —Ltog “of +e Git Let tof hea g) INTEGRAL CALCULUS MADE EASY iv) Let Ay log] 2/3 | ye te r42/3 Bee2 = tog | 2=2 | + = 39 '°8 3x +2| * 1 ¢ tsf—t. te Sear -* 1 1 “fw INTEGRATION BY SUBSTITUTION—1 99 Example 21. Evaluate the following : of wayree ti) J pg ode aio J Solution. (é) Let 1= [ fd toy J joy (Multiply and divided by 3] Put -a' 3 za’ +e zea 100 INTEGRAL CALCULUS MADE EASY 3 +e. =x] ae Wd +x log] —~——* Wa2-x +e = te loy =22 | 1+ v2x 1 Vax Example 22. Evaluate the following integrals : I a8 © | rams * voy _ 33 . = Gin | eae Cd ee +e. Solution. (i) Let I= f ——1 ae ade sec? a "Jone (Ge sec? A -tan? A = 1) 1+tan? x+5tan? x =| ee 1+6 tan? x Put zztanx 3 de=secixds X46 a INTEGRATION BY SUBSTITUTION—1 101 tt, afta 64 25. —e. tant) 4] 40= 26 ewe (eles tan" (62) +6 = yg (Beane) +0, fs z=tanx] ii) Let 4 = se i : {Add and subtract 1 to the numerator) x -1, 1 “(5 +e =a) ae ef[@oDat+D 1 on pee -/[42¢ Fa dx ts a?-6%=(a+6)(a-8)) = Jet-Ddes] Thy ae [by sin ft coir den Lean Es, of estartese, 3 vs 3x # (i) Let t= J pope d= 9f yey oe 3 a =a) Tragy 2 [Multyply and divided by 2 Put zea? = de=dede ap 3p 2 ‘ 13) gor #-as Ta 2 1 42 dz B aig den ita 4 “irae +2| [iy sing | rts nv Ee] tan? a2 way lea]? * tant (Ba) +6 = jg oo" (e") +c ts rex) (uy Let T= past. as +4 eed 6 “{(S-3 wa) #8) 102 INTEGRAL CALCULUS MADE EASY «furans alge * [ =x-5.(3) ian (3) +6 a1 $san-*(2) +e. Example 23, Evaluate the following integrals : — 1 ‘iy { ——1—— © | Gare meat ye «of (842 cos? x) = stan! Sa¢] 7 i ca J x74 3dog "1 L 7 x +508" x) ” (Dividing numerator and denominator by cos? x) Solution. (i) Let I= aa 1 2 2 OOS Ede = f 800 sin® x | 5 cos? x 4tan?x+5 cos” x cos? x Put zetanxy = dz=sec*xdx de (az +5) a “J B.3 5) * . 1 Lyi [iy sing f preg dees tan Ene] 1 et z= tanx 25 } (i) Let T= [ ———,— .dx [Dividing numerator and denominator by cos? x] 3+4+2cos* x a cos? x sec* x de=f —C* | 2cos? x Baec? x42 cos? x cos? x 2 2 =f __ sec * dee f OF ade (1+ tan? x) +2 8+8tan? +2 (> sec? A-tan? A = 1) INTEGRATION BY SUBSTITUTION 103 sec? x 54+3tan? x Pat zstans = dzssectxdx 1] iat ang] [=] 3 [0 z= tan x] _ a ix (iii) Let Is Jaan . =| equ? . Zed] Example 24. Evaluate the following integral : 1 o ode w { x+8cos" x Sie —_ 1 143sin? x+8 cos" x (Dividing numerator and denominator by cos? x) «dx, oJ 1+ sin Solution. (i) Let t= f 104 INTEGRAL CALGULUS MADE EASY 1 cos” x Ssin? x | Bcos? + cos? x” cos? x sec? “J (eee? r+ 3 tan? x48) sec? x sec? x Geant: dente *) 7 (1+ tan? x +3 tan* x +8) (9 +4 tan? 2)" [ly soc? A= tan? A= 1) Put z=tana = de=secade * Is Jaw E 2 ly z= tane] (it) Let ads Put = iaf_1_ i ae “sl ae [Prin f zy teepian ‘ G)=( INTEGRATION BY SUBSTITUTION—| 105 Example 25. Evaluate the following integrals : i I D| > - ae i) | —— . de o] 15 - 8x? “ j Vi6x? + 25 fii I 1 «io | rare oof Teme wip f he de. x 7-5 1 1 1 ie al ea 8 = ae 2 Git =f pee det 16x" a+ J og| + = jhe =A bog] + = oe 1, | 4x+ fix? +25 = ghee 7 ey 7 Jog m.—log n = log 2] 106 INTEGRAL CALCULUS MADE EASY = Piog| 4x+ f16r* +25 | +c. as L 1 1 GiyLet =f eeu tea) pow jer = dfoe| 4x fies? =9 |] - 2 toe 141 +6, [: logan log n=I0g =| = flog| 4+ Ji6x? -9 | +0. [where se= ey -2 toga] e (v) Let ts] aay Put ze! > dewetide 1 ef[ay-e 1 “Sera * INTEGRATION BY SUBSTITUTION—1 107 Example 26. Evaluate the following integrals : . 2 a sec? x wf ir oe ait J —; -dx tio) f J2=% -ae x? . 1 of ae wo | ee Solution. (i) Let I= Put 2=2' 1 = . a -d2=2. de = dz=2log2.de = jog dz . Jets s I= aw (ioe? dz 1 nga! Tare & [» using f 2) iteyeee be vain 2 Tog2 @)+e= ep sin (2) +e. . see? x Gintett= | a a Put z=tanx = dz=sec?xdx 108 INTEGRAL CALCULUS MADE EASY = Isf mor [rain Jae ein ee +4 loa | 2+ Jz#+9| +e =log | tan x+ tan? x+9 | +e. f: z=tanx) tiv latt= J pe dee f z Put z=e# = desSetde 2 ddewe de 1 1 \-slasa-* dz elas =t)—at= © 124 (2? -9/4) © _— 1 aca “al 2) ide = +«| 2 afiu 2-3/2 1 igg| 227 = i2'{a@7m|'!8| 25372 |*°> a6] 524 |*¢ 1 fae =3 =p el FF +e by: ane (ytet T= f [2% ae ate =f ex ae {On rationalization) ase” Jane -f (a-x) - dz=me*dx = =dz=e*dx. 1s] cu girs [prog pty aeot xo fe [ol 24{F=1/ +0 et feF=a| +0, te zee) (Multiplying numerator and denominator by e*] = log 110 INTEGRAL CALCULUS MADE EASY Example 27. Evaluate the following integrals : ofa a las MO) log v? ~5 * iid) f ; 2x? 2x8 Solution. (i) Let I= dee { 2F uments’ Jae 44 (xh? Put zext a = dendvtds = FS ashds “ -[—e a “ “F (dz) 4 a 1 ing { —2.de=} tan? =e [Py ose f ye ep ee] afl (2) e2ibant (2 21 (!)uant(s) ceed} +0 a a = | x (log x)” =5 Vlog x)? -5 Put zelge 3 deo dx 1 tf ogee =| eee =log| 2+ (2*=6 = log | log x + yflog 2)? ~8 | +e. [ye z=log x] (ii) Let By using j roe ees I +e a a at Gi) Let iS dr | Teta ts Put z=a* => desatlogady =» —)_ desat.de ioge a =| lige 1 1 . “gal Jeo [pesine eo sintztce sit @) +c. [: zea4] ioge log a ae -de=sin =4¢ a INTEGRATION BY SUBSTITUTION 1m Example 28. Evaluate the following integrals : . J sin x ay sec? x ) rea to | ard Solution. (i) Let I= J Teed act ade Put = smi Tere ees ae | 2 zt +c, 2] reyet-g 1, |= {427-1 — Fog] BA —* 1 1 == 7 log tos~ 5 log +c z =~ Efloe| 22+ f4z*=i|-tog12i] +6, = Flog | 22 + faz? -1|+3 log 121 +e, =~ 3g | 22+ faz*= alse [where:e=e, +2 tog12I] Ls reece] =~ Blog| 2008 2+ feos? == 1 cn sec? x te =f Jisrunte Put = z=tanx = dz=sectxdx tn] peg are f hie. de TF fiese? OS fies [» using J iF dx =log| «+ fata +4 +e 12 INTEGRAL CALCULUS MADE EASY 2log|2+42?+16| +6 = log | tan c+ ftan? 2 +76 | +6, L; r=tanz] EXERCISE FOR PRACTICE Evaluate the following 1-sin x (og 2)* cos x ~ sin x Acsin © oy cos x sin x Ly veo e 2. f SRP ae S) "Tyan 2 sin x cos x * “ J issn Jxme «fies 5.) Tssin 2x 6 J xsina’ de sin. (tan x)? 1 ede nw de x J (a+ beos x 8) Tee 9%) Jireos ax L+sin 2x eed 10 J eran? uf aie sy sin (x +a) 13, f tantards uf Rose 16. f sin’ soos" x dx 14. J tanSetan2ctan cde 18, f sin’ xem? x dx 1 1. log | x4 case 1 +e 4, - log (1 +008 x) + ¢ 5. tan“ (sin x) + ¢ —_1!__,, (tan vt +6 Bat booe x) 4 1 9. Fy low leosee x — cot. +e 10. Jog | x4 sin?x lae 11, dog Ce +e) +e 12, log | sinx +c0s x1 +0 13, tan? x log tee x1+ 2 14, 4x—a) cos 2a + sin 2a log | sintx-a) b+ 15. -log | sine +tanz | +2sinx+c 2 jlostsce 9x1 — 5 log ave 2rt— log le x1 + 2 jgg| #2 8 12 cox@ 18. Psi ae 20. +e. 3 Integration by Substitution—IT 3.1 INTEGRALS OF SOME FUNCTIONS CONTAINING A QUADRATIC POLYNOMIAL valuati 1 a 1 Evaluation of f pads. J Pc ds le and [Bt ae jax? +bx+e ‘The quadratic expression ax? + bx + ¢ may or may not be expressible as the product of linear factors. In case, ax? + bx + cis easily reducible to two linear factors, then these integrals can be easily evaluated by using the method “Integration by partial fractions”, which we shall study later : Thus ; (a) If ax? + bx + c is easily reducible to linear factors, then either the method of this section or the method of integration by partial fractions can be used. Out of these two methods, the method of integration by partial fractions would be found more convenient and easy. () If.ax? + bx +c is not easily reducible to linear factors, then the method of integration by partial fractions is not applied and the method of this section is the only choice at our disposal. 8.1.1. Working Rule for Evaluating | wipe: Step I. First take a common and make the co-efficient of 22 unity, or by multiplying and dividing by it. Step IE. Complete the square by adding and subtracting the square of the half of co- 2 efficient of x ie. (J eoent of +) . Step Il. Express (ax? + bx + c) as the sum or difference of two squares. Then to inte- grate. Step IV. Use the suitable formula from the following : 1 fcotie ae | opty ar ona J 3.1.2, Working Rule for Evaluating [ x 1 ay, vax? +bx+e Stop I. Make the co-efficient of x? unity by taking B out of the integral sign. 113. 114 INTEGRAL CALCULUS MADE EASY Step H. Complete the square by adding and subtracting the square of the half of co- efficient of x. Le. G coefficient of *: Step III. Express the integrand as : 1 L 1 — de =~ [ La. Jam ial [erarap Step IV. Then to integrate, use the suitable formula from the following ; 1 1 [lz tn | og aes 2.1.3, Working Role for Evaluating { —?="9— . dx, Step L Put pe +q)=1[-£ (ax* +40 +0)] + and find the values of A and u by compar- ing the co-efficients of x and the constant terms. Step IL. Given integral takes the form as : pe+q . Pact at ebere 8S Sry pees MHL atv peee Step III, Evaluate the first integral by putting z = ax? + bx + c. ‘Step IV. Evaluate the second integral by using the method discussed in Article. (3.1.1) 8.14. Working Rale for Evaluating [ Jenne Step I. Put (px +4) = ing the co-efficients of x and the constant term. Step IL. Given integral takes the form as : prtg 2ax+b 1 vde=d edt pe oe, \ er Gea 8") Gee Step IIL Evaluate the first integral by putting z = ax? + bx + ¢. Step IV. Evaluate the second integral by using the method discussed in Article. (3.1.2) Lat +20] + Wand find the values of and w by compar- 3.L5. TYPE : Integrals of the Form | z+ ——. dx. (Note. For Working Rule Please Refer to Article 3.1.11 SOME SOLVED EXAMPLES Example 1. Evaluate the following integrals : ade dx | acess | ara INTEGRATION BY SUBSTITUTION—1l 115 a 1 to | Seca ® wf vasa 1 ©f ggce * oll a 1 Solution. @ Let 1= | 33975 -# 1 1 , =5! T"FT7B) dt [Take 9.as common from the denominator] le ~Gx48) Add and subtract * to the denom. =f L dx 1 y 3 (#403) : (Jevett ot =} =< Gi) Let [take -; as common from the denon] Add and subtract 3 to the denom. 1 . ie (3 coest or) =i 2 9 116 INTEGRAL CALCULUS MADE EASY i Bx+1-V8)| 6/2 | Sx+14J2|°~ ai (iD Tat T= lana * “Hl ew ‘Add and subtract 2 to thedenom. 1 yt (ecerers) <2 (+3 Yew 1 Puts+ 3-2 => dx=dz lg . L re? p «-{—> By usi ay edz stant te sot slaup-e [» ina | ao a a | (Bester Sd (et -Ox4D-4 nr =P -@ Put (e-Dez = dredz Gs z2@-D) tv) Let L =-J ¥aer-8 * : 1 “Add and subtract 16 to the denom.’ -[{-— 1 ° J (x? +8x+16)-21 © v (Zovenot x) =16 1 :-l eae Put &+4)=2 = dreads 1 . s sey By using 27 —(J21)? [ Sy -— hog z-V2i| 2.J21 | 24/21 1 44-21 =- at = 4) sar alte ty zee) 1 (wi)Let 1 4x? 4x43 118 INTEGRAL CALCULUS MADE EASY Add and subtract 3 to the denom. 1 es (Jeoem.or) =} Bamps 2. Evaluate the following integrals : a 1 oJa4u- wfois-: oy . i a — dee waa J x 4448 wo f at ex-2 - 1 Solution. () Let = f =~ - Add and: subtract Lig the denom. Jaye ae eyo" INTEGRATION BY SUBSTITUTION—II 119 Put (x-2) <2 = deed tf -1/2 tao (2 Jee V8 /2 tan (2554) 60 1 1 1 = re Se] Se z 8x° + 132-10 J, 2) ale ale (ii) Let dx ° ‘Add and subtract! (2) tothe denom. ” (3 evettots) -( a) 2 6 120 (iii) Let (iv) Let INTEGRAL CALCULUS MADE EASY x 44x48 ‘Add and subtract 4 to the denom. -[ao te : : (2? +4244) + (8-4) © v (Zevemer +) =4 =J area r¢2ez = desde [arsine J dx= Fant 5 «| z [fe z=x+2) 2 ‘Add and subtract (3) to thedenom. © Goal (ah INTEGRATION BY SUBSTITUTION 121 Fro 2x-1) 4, *| ben |" Example 3. Evaluate the following integrals : 1 x ey ee of re ® w J e* +607 +5 | a oa = A 1 ao J a rie? ton J xara I wf aap oh Letl= | ———— Solution. (i) Jao a Put az = INTEGRAL CALCULUS MADE EASY (di) Let Put Putz4+3-y (i) Let I= J d= Lean Ese] ae [y y=z+ V2) ty 2=27) t ° ” (Jooettof x) =9 1 -J (2? +6249)+ 6-9)” [" and subtract 9 to the =| dz 1 “lara * @F = da=dy t=] 3 ~ 55 8| 235 2(2) 1 “4 “ore [arsine f 2S 3 4e 2+3-2 24342 +e te Lgg| et 4 =| +e. e485 x x ax" - 1627 +11 vee J ae aoa INTEGRATION BY SUBSTITUTION—Il 123 Put atez = Qedeadz = sdew 3 de = 1 1,\)o3 1 “| gece (2e)*a) risen & 1 1 OF -62+9)+(3- 9) (" subtract 9 to the 2 2 ° (Sever ors} =9 Put 48 | 2-3-4/ V8 = no yae8 4882 -344/3 | °° fe yas-3) Blog| 2848 aa 468 °8|P—a44ra|*° a _ Vi jyg| et 3-4) | 48°") Yas? 3/344 |*° i -f—e (iv) Let -foaqp-* Put x2*+lez = x"=z-1 = neh) deede 2 ME dead 124 INTEGRAL CALCULUS MADE EASY Add and subtract 1 o the denom. & 1 vod " (Jeoen.or x] oa [vo y=z= 1/2) n 8) z-1/241/2 sa 1 | tee x 41-1 x +1 +e zax"+]) 1 1 4 n = 4 tog|—— 0 —1_ x(x5 +0) Pot x8+1ez > xezel (Let t= f = bxtdeade = bi tdred: 14-1 1 1 tartan 1g) oar a = 2" "a-0 © 1 ou ipl. I= serpy es ‘Add and subtract ito the denom. 1 po. zs (Jeoer.or =) 5 INTEGRATION BY SUBSTITUTION—II i a Ly | z-1/2-1/2 = 5 he see ecet be yse- Va} te z=x54 i) Example 4, Evaluate : cos a © | Sayetaness sin? xtdsinzse of con (let Te 08 x Solution. (i) Let I= sin? z+4sine+5 Put sing=z => cosxdr=de 1 * tf 2+are5 1 ‘Add and subtract 4 to the denom. 1 ___ 2 (2? 44244) +(5-4) ae pooethof x} 1 “J Gare? * Put z+2=y = dze=dy 1 . 1 “ly as s 1s] ae dy [By ins f yee £+4] stant (2) +e tan"! (2+ 2)+e = tan (sin x +2) +0. > ysz4+2) z=sinx) INTEGRAL CALCULUS MADE EASY let I=] 2 Put az => a2dvedz = xtdy x+a@ =t|+e] [vy z=x4] 3.1.6. TYPE 1: Integrals of the form : [ dx. 1 yax?+be+e (Note. For Working Rule please refer to Article 3.1.2] SOME SOLVED EXAMPLES Example 5. Evaluate the following integrals : 1 1 i wds i) f —+__., an ww f jee a a 1 - I — et =z: Gio J Kao to) f sams te wf de wo | Se fe—ayte—6) ° OT Yast sb? 1 Solution. (i = . lution. (i) Let I ls de -)§ St Vie 22+ +40 1 J —s J Ke — 9F +(/8)? Put x-1ez = dred ‘ “Sara [praine otg aetna] : (3 coeff, of x} [* and subtract 1 to the x INTEGRATION BY SUBSTITUTION~iL 127 =log|2+ fe" +08)" | +e =log| x- 1+ (= DFS | +e =log | e—14 P+1-2x+3| +0 bs ex) = og |(e—94 = Be 4 | +e. @ 1 a tle i aaacg ee! Gee Add and subtract to thedenom. log +e. (2-3) + (eee (ii) Let 7. ar= J 1 de Of \7-6x-x7 Fret +63) | 1 ‘Add and subtract 9 to the denom. -J Teed 1 2 749 -(2? +6249) (Beoemat x) =9 1 16-43 128 INTEGRAL CALCULUS MADE EASY Put 243-2 3 deade 1 1 az A Is wde By using -dx= sin’ (2)+. I qehes [prin poe aroun : (3) =sint (2) +0 saint S49 4, fy z=x43) ; 1 1 Gv) Let | Ea F ‘Add and subtract to the denom. 1 1 =z) od > Wea fGen) a alae be 3 Put atgre = dx =dz e wal aR * [pain ra ers +4 +c +e INTEGRATION BY SUBSTITUTION—I 129 1 1 Let 1-f ——— .«- | —=——..& ® Syne \ ere 1 ede exo (a+b)? (a +b)x + —— 7 +ab Add and subtract (4 b 2 2 E coeff. of 3) = (34) 2 2 2 ) to the denom. [pains Tro tele feat I +e 2 (a-by wees) om) ieee 2 z a 3 = log (x-2 =), es — ee +c = (= o48) fe arb) +ab = log (: -2e)s [(x-ae-8) | + «. ' 1 LL 1 __ (vi) Let 1° pave ali 3° 3 130 INTEGRAL CALCULUS MADE EASY ‘Add and subtract 2 to the denom. 1 ? 25 (Ze-emors} "3 [ms asin ar stg PH fo] 3 = igine|e (8) +e = 7h («+8)+ 59 = flee (2+) + fers +2 ota (<+8)s [PFat| 0 Example 6. Evaluate the following integrals : (ta ©) Teac (ity f rar of ma * Solution. 0 Lett = f rats dew | Tomer ae INTEGRATION BY SUBSTITUTION—It 131 Add and subtract to the denom, 2 Ss G coatt of x} ai 2 16 1.4 = yy sin? Gr-D +e. aj (ii) Let 1:f —— \8+3x-x 132 INTEGRAL CALCULUS MADE EASY = sin! (4) +e sin /273/2),, a Vasa emi fe=d (8) (Ho Tet [ae ‘lee ‘Add and subtract 4 to the denom. 1 2 = ode 1 Seas * (geoeror} =4 -J—=.« Ve-2"-2 Putr-2=2 = dexdz =| ae [prains | pty aeot log | 2+ V2*= (2) | +e log | +422 | +e =1o¢ (2-24 2-2? -2| +6 te zex-2] log | (e-2)+ fe*=4r+4—2| +0 ~log | te—2)+ fa? | + i 1 toy Lat = | Toe | 1 ‘Add and subtract 36 to the denom. = ode, 1 " | ees ° (Jovett ot) =36 ‘lyre Pute+@)=z = dz=de =| oe [nin phy deat 0 =o] =og | 2+ fe | +0 | = log |(2+6)+ x +6)*—25 | + [v 22x46] = log |(2+6)4 fe? +1224 11| +0. INTEGRATION BY SUBSTITUTION—iI 133 Example 7. Evaluate the following integrals : of cf rope ade io f jean de wf hae -dx wi f (vit) { BA de wit f isin’ +4 sin? x-2 (a= x*)(9 +(sin! x)*) ° Solution (Let = [TE de Put ez = bedreds ‘ =[-4 e-f He “ 1=f zt J mun ‘Add and subtract + to the denom. Tee | Game a =[ se ed Put 2+ 529 = de=dy 134 INTEGRAL CALCULUS MADE EASY cos Wit 1-f ———— a ce ain? x-2ein e- Put sinx=z = cosxdx=dz 1 le) ———— .& J ye? -22-3 J 1 fm and subtract 1 to the: _ ade hb 2 (2? -224)-3-1 ° (J coett of x} =1 1 “J Ye-r-a® Put z-1l=y = dz=dy el qehge 4 snc tye To yo xc“ — a’ og | y+ p*-2 |se=tog|y+ yF=4| +0 =log |(e- D+ Yle-DF-4| + te yee-H = log | (2-4/2? 2241-4] + 0 log (e- D+ fe? =22-3| +e = log | (ein x -0)+ Jin? x= Bain x9 “ . sec? x . sec? x (iii) Let [SS ¢l Ghar * Put tanze=z => sectxdrade “loa [pring shay seo Fao +2 lat +x =log|2+ fe? +0" | +e=log|2+J2%+16| +e =log| tan x + ftan? x4 16 | + bt: z=tanx] +e [fy 2=sinx] x x Let I= | J = d= | Sep ede | (we) at Ite lo Put wez = O2dxadz = dx = 5 de i=] = =G#) -z INTEGRATION BY SUBSTITUTION—II 135 w(t cn*(}se sing! (3) +e, fe z=x3] lee * Put az = dead: . = 1 = 1 . . “|e & laa * Add and subtract 4 to the | 1 “J BricGiedeeh) =f que ae Yo- G49" Put z42zy = dz=dy colts estes it a tv ysze2] [. z=et) (vi) Let = jattde de = Vi de=3 de 1 2 * 12] Tour G*) 2 By using deosn Eee] lye * 136 = pan 3 sin 2x (vii) Let T= de J ein''x+4sin? x2 Put sin?x=2 = Ysinxcosxdx=dz = sinQxdr=dz 1 Joho 2" +4z+4)-2-4 2 =| 40 a 1 o =| SS ' J 2442-2 1 “J Jara -6 Put z+2=y = de=dy 1 . 1 os 1=f oa dy [rain Paws =log| y+ 197 =C0F | +e =log | (e+2)4 (2427-6 | +6 | INTEGRAL CALCULUS MADE EASY te ze x¥} t+ 2sin Acos A =sin 2A) ‘Add and subtract 4 to the denom, 2 “ G coeff. of) =4 te ysz+2] = log | (2+ 2)+ \z* +4z+4-6| te =log|(2+2)+ 2 +42—2 | +e = log | (sin? x+2)+ fsin® x + 4sin® 1-2 | +c (viii) Let T= f 1 = ade J [9 +(sin™ x)*] y1-x* Put sintx=2 = 1 fa=23)19+(sin 7) | wdzadz 2 INTEGRATION BY SUBSTITUTION 137 1 4 1 Taq) =f{—L—.a@ | Byusing wde=log|x+ Vx? +a? |+c lone J Tet | | = log | 2+ +9) +e =log| sin“tx+ Jisin-" 2 +9 | +e. Ls 2esintx) Example 8, Evaluate the following integrals : i) f JTF conecx . de i) J focox=7 « dx. Solution. (i) Let I= f [Trcosecx . de wf fori te= | RE? . f (i+ sins) —sin x) ~ sin x(1- sin x) [On rationalization) Le (a +6)(@-b)=a?- 27) J sin x-sin? x =f ee fy sin® A+-c05? A = 1] Jeng -cin x Put sinz=z => cosxdr=dz 1 1 Wee el Tay Add and subtract 3 to the denom. 1 vii ‘ (J overt of x} =i Put 198 INTEGRAL CALCULUS MADE EASY = int (GP) te bs yee-19) sain? (22-1 +e =sin* (2 sinx-L +e. [e z=sinx) Gi) Let Is | fecz-1. dex 4-1. dus | = =f (= coax) (1+ c0sx) {On rationalization} Y cos 2 (1+ cos x) 7 =[/32"3. & fs (@-b)(a+b)=02— BF) cos x + cos" x - | —Be ae (sin? A+ cos? A= 1] eos? + con x Put COs e= Zz = -singde-dz + singds=-de 1 [pee ‘Add and subtract 1 tothe denom. Spo ” (Jeoettors} = hay (+3) -(3) pu (r+3)=2 = dendy ls By using j 1 ide = x+\x?-a" [4c pag? beer PO ees (e+3}+ie%az| +e == |( ooss+3)+ cos? x +c08.x +e we [: »(e+3] +e. {2 z= cos x] INTEGRATION BY SUBSTITUTION—I 139 __Px+q 3:17, Integrals of the Form J P= — sdxand J Jerome (Note. For Working Rule Please Refer to Article 3.1.3 and 3.1.4) Seamples 8. Evaluate the following integrals : de lores Tuas ad fe 4x43 gee -3 4x41 tid | ee a ® tin | ogee Py 3r42" 2r . of Se ads wif wea 2-3 Solution. Let T= [ sag og - Let us write (as-9=1[ Leas? +4045)] +0 = Qe-F=A (Gx +4) + oD = Qe- Ga Oars Gh+ Ww) Comparing the co-efficients of x and the constant terms, we have 1 2=6A => aeg and —3=4hep 1 = -3-4i2-3-4(2) 4__8 . we-8-55-4 Putting the values of 2 and yt in (1) : => (2-9-3 644-2 . 2 Ge+4)- 13 3 I= j Bx? 44245 4y 6x44 13, “SI (Set 44x45 Ox? 44a +5 Ay _Gs+4 -8y 1 “34 8x7 +4045" 35 xt 44x45" dx 140 INTEGRAL CALCULUS MADE EASY 1, 38 = Iegh-Zh AB) 6x+4 fT Bae Now =) gregg [: J jo? lagitot+e] = log 18x? + 42 +51 +c, (8) 1 “ee ‘Add and subtract 4 to the denom. fl y4 : (Jevett or) =4 see pws [py using oA) = 3 fog tae! sae +61 se B[ Feet B2,] [Using (3) and (4)) 1 3x+2 "3 log 1322+ 40461 +4 307 an ane a i = Mog | 8 sare 5 1-37 tant Be +e Gi) Let de Is ‘Sats 4243" INTEGRATION BY SUBSTITUTION—I 141 Let us write: x-1-2 [ioe ~42+3] +e = x= sh(Gr-4)+ = Gir 4n tH o(L) Comparing the co-efficients of x and the constant terms, we have 156k = aed 4 1 and -le-4i+y > wenl4ahe-1t Gao 5 Putting the values of 4. and win (1), 1 1 (x= Ds | (6x 4 3 1 1 2(6x-4)- = 1sf S____3 og (8x? - 4x +3) = w(2) Now {5S wdx= lag Fae] = I = log 132? 4e 431 +, (3) and wlecn “Gg “ey ads Add and subtract 5 to the denom, 1 ads : 24,8 0-3) s G =4 ar) 9 + (Jeoettore] =F 142 INTEGRAL CALCULUS MADE EASY (4) ies = wat ~4r+3142 een = ot togise? — 4e+81-—L tan 6 a5 (iii) Let fot bt 2 Let us write : (2a) 0n[ Zc? te 209] +8 = (2r—3) =A + D4 y nf) = (Qe-3)=2e4 (G+ Comparing the co-efficients of x and the constant terms, we have 2eM + del and -3=3h+p = =-3-38h=-3-3) = we-6. Putting the values of & and jt in (1), (Qe—3) = (2x 43)-6 (22+3)-6 (x? +3x- 18)” =f 8 ae 0 or x? 432-18 xe va 18 = Isl, -61, 2) 2x43 =f te ak levee I, = log Ix? + 8x- 181 +0, (3) [- 4 FO) ae lagifo+e] Fey 1 nd he) Frac INTEGRATION BY SUBSTITUTION—II “J oe o z£-9/2 2+9/2 . Iog| ey <. From equation (2), I=1, -61, 2-3 +6 1 = flog Ix? + 3x -181 +e)-6|gbe 2, |x = log Ix? + 3x- 181 +e,- = — 6e, * 1 gel ye| 2, | x3 log 1x2 + 3-181 — 2 tog| £=3 , axe anit t=) Ah ae Let us write: ax + 1=a[Letsacen] +4 = 4x4 Lad (2434p = 441 =2ar+ G2 +W) Comparing coefficients of x and the constant terms, we have 4=2A => A=2 and 1=Sk+p > pol-34=1-3(2)=-5. feed ld) [Using (3) and (4)} where :¢ = (c, ~ 6¢,) (1) 144 INTEGRAL CALCULUS MADE EASY Putting the values of) and jin (1), “de + 1 = 2204 3)-5 1-4 U2x+3)=— -5 rrr a _(@x+8)_ 1 -?f aed Bearea a = [=21,-5l, (2) 2243 Now =f ee = 1, = log Ia? 43421 +c, (8) [« Fie) de=ogifsh ee] 1 and 4: | ma ‘Add and subtract 9 to the denom. “J 48e4 , e-3)* 1 7 9 ($ coett ot =) 2 Soe (=+3) 4 Put aepee = dx=dz INTEGRATION BY SUBSTITUTION—II 145 xt = 2[log Ix? + 3r +21 +e)-5 [he ns +4] (Using (3) and (4)] = log 1x? + 9x4 21 + 2c, ~5 log | = xt s Tn og Ix? + 3x +21 - 5 log | **> where : ¢ = (2c, ~ 5cq). (vy) Let T= Se Let us wie ar =2[2@ex-24)] +n Qesk(1-2Qx)+pa-2arehep (1) Comparing the coefficients of x and the constant terms, we have 22-% = de-l and O=hep — pa-k = p=-Cl)=1 Putting the values of ?. and pin (1), Be=—(1- 2x) +1 ~~ 2x)41 Beye [ogee = => wl) Now = Lelogi2+x-el +e (3) [- Ei aden logic] and bef goa ade ‘Add and subtract 3 to the denom. we (h y 2 « (Jeoemots} = 4 148 INTEGRAL CALCULUS MADE EASY atx ately i 1 1 [by oni f ag ng 8 3 2*?|" 1 1 log 2 +e,= gle 3 [» zax-J] a, 2 x+l => To +e, (4) From equation (2), I=-L+h ie, ene Eh gr 4] (Using (8) and (4)) anlog I+ 2-31-0445 + tog| =*2 +1 Ale =-log 1242-291 + Zilog] +2 ae, where :¢ = (¢)—¢)) wile I=) aa Let us write : Weenie => xeA(Qr4 1) +p (D) = xeBe+ Orn) ‘Comparing the co-efficients of x and the constant terms, we have Le ded and O=Aep 3 ps-k 3 pend Putting the values of A and j1 in (1), x=}aren-F dtas+n- * 1-f25 ode (xh +x4+D 1 ae+h 1 1 =>5/ = —.-de-5 | = & 25 ext 24 xeztl 1 L = Ist h-$h (8) INTEGRATION BY SUBSTITUTION—II 147 ax +1 Now LeJacn: : [LO ae = log Ist+ x4 11 40, (8) [: J Fin Ge =boalflaire 1 and 4-J apm Add and subtract + to the denom, 1 vod + [Zeoeft. = G os] 4 Put sees = dx=dz 20 ete =H Bate 2,0 (deed =e) ro ++ From equation (2), roi 1 gh Nie le flog tte st ees-4 (Using (3) and (4)] =i 4a AL tant (2242)_1 = glo istered +t 1 Fy tan (44) 5% so plogistexe to tan(BEt) oe 1 1 where:e=([Zei-Jes). 148 INTEGRAL CALCULUS MADE EASY Example 10. Evaluate the following integrals : _ 2x-1 Gx-2 ae ae 2x+5 i, e'aetd (ii) @v) az, ® [ea5 a | SE open a won f Stet tat] oy, x +3x42 -x+1 ' Qe-1 Solution. () Let I= | sar - Let us write: ae-12a[Zcaetsae+n] +H = Qe-1 = Mar +2) +4 = Qe-1=dde + (Qh +W) ‘Comparing the eo-efficients of x and the constant terms, we have 1 2-4 > ARG and -1s2+n > wend-mhan1-2(5}=-2 Putting the values of i and pin (1), tag ean 2 A ae+2)-2 I J Qa? +2c+1 ” 4r42 dr-2f 1 3 2x? 42841" 2x? 420417 = re} -2% 4x+2 Now ne) gt eeeed” wlog (B24 2x41 te, (8) [: fe fe) of) wl) -e=ogifcol+e] INTEGRATION BY SUBSTITUTION—I 4149 1f4 42 =2( 5) 7 *% war (2222) 40 1/2 stan (2x +1) +e a) From equation (2), 1 1=5 4-2) ie, 1-3 og 12x? + 2e +11 +c] -2 [tan (2x + 1}40eq] [Using (8) and (4)] = dog lat42re tle 2e,-2tert are D~2e, 1 = 3 beg 12x? + Qe 4 11-2 tan! (2x + te where se = (3-261), 2 Gut 1-f Be adre & Let us write : be-2=0[ 2 cae? vase 0] ru = Sx=2=)(6x+2)4 0 (1) = Gr-2=6hr4 2G Comparing the co-efficients of x and the constant terms, we have beh > aed and -2eQhep = p2-2-2A =n Putting the values of and pin (1), t-22 2649-2 Sex+a-4 1 | Grasp ora 5 6x+2 ul 1 =f ——.«-=] =. (Sx* +2240) Saige a INTEGRAL CALCULUS MADE EASY ie, I= 1, 122 fog 13224 20411 setae (Se} 6] 5, u eh-gh 6x+2 Bx 42r41° slog 13x24 2x41] #0, (2) - [£@ ae -B) [: Fo odx = log lf(z) +e ‘Add and subtract 2 tothe denom. Jf yu (Beware) -2 (Using (3) and (4)] P x == 5 tL tan? i 5g log Nox? +2r+ Ul + Gr zm (2) zo INTEGRATION BY SUBSTITUTION—II and and Ox+5 (iii) Let Ja: Let us write : dx aeener[Zit-2-2] on => Qe +5 = M2x-1) +p > Bre 5a Qx-k+p Comparing the co-officients of x and the constant terms, we have 252k = Asal Rew = WeSed=5e1e6 Putting the values of 2 and i! in (1), (2c +5) = (2-1) +6 ya f C=B48 wax-2 0 Qe-1 =[ay esa e = Is1, +6, Qx-1 Now eoag £@) = fore sf(—. og Ix?@=2= 21 #0, (3) f J Fo bf aoa 151 wD) AB) d= lglfco lee] Add and subtract + to the denom. .f2 v2 - (Bewatots) = 152 ie, +. From equation (2), INTEGRAL CALCULUS MADE EASY I=1, +61, Te flog 1x? 2-21 +¢,)+ 6] 410g] 2-2 | +6, [Using (8) and (4)] +8] g tel ed x-2 log Ix? 2-21 +0, + 210g |= | + Ge, x-2 = log 1x?— 2-21 +2og| 2 te where : ¢ = (c, + 6c,). 2 Goytet t= [ $244 ae (Dividing the numerator by the denominator] xx stoz) @4es10 +xfuk a+ 24d nf) (2) Comparing the coefficients of x and the constant terms, we have 22 => hal Ath poledelere2 Putting the values of 2. and yin (2), (Qe 1)= (x= 142, (2x- +2 he)" 2x-1 = Le] oy de+2f 5 = I=], +21, (8) 2-1 Now u-f Foe log Ix? x1 +6, A) [ C2 = togi 0] INTEGRATION BY SUBSTITUTION—II 153 ‘Add and Subtract, ; tothe denom. Put From equation (3), I, =1, +2, slog Ix?-xl +0, +2 [me (Using (4) and (5)] a-1 slog Ix?-x1 +c, +2 log + 2cy log Ist 21 + 20g |=") + (6) where : ¢ = (c, + 2c.) From equations (1) and (6), we have L=x+ log [221 +2 og| +e. 2 (v) Let Ie xtbet8 | x 43x42 2x41 l= 1+———— | . de J wh) (Dividing the numerator by the denominator] 154 INTEGRAL CALCULUS MADE EASY > ts J tder) Pt ax x? 48x42) 27 4524301 +3x42 2 x 43x42 = I=xel, oD) ee on+4 2x41 Now he] yan —_—_—_— Letra: d Neer aa[ Zot vaes2)] eu = Qe +1aA2x43)ep (2) => Berle Rer Shen Comparing the co-efficients of x and the constant terms, 22% = hal and TeSA¢p = pel-3421-3(1)=-2 Putting the values of 4 and yin (2), (x +1) = Qr43)-2, . Qxre3)-2 ~ 1) Gt yare2) 2r+3 1 =f 3. ae-2f 1. - hm) Fyae8 Suen = 12-2 (8) 243 Now =) Fyax+2" slog Ia?+ 3x #21 +e, 4) [- Oe are lagircore| and -[=.4.-e b= ayes ‘Add and Subtract ¢ to the denom. : (Jevemor = INTEGRATION BY SUBSTITUTION—Il (5) From equation (3), I, =1,-21, atl * I, = log Ix? + 8x4 21 +e) — [| xt2 +e; +] [Using (4) and (5)] = log Ix? + 8x +21 +6 — 2 log | = =. eq = log |x? - z a log |x* + 8x +21 -2 log zea|*° wd) where : ¢ = (c;- 2c) 2. From equations (1) and (6), we have Tax + log Ix? + 3x4 21-2 log | © (vi)Let I= jae tx+1 xoxt1" Please try yourself. (Hint, Divide the numerator by the denominator, we have 1 f (1 fAns:x+log le —x4 1+ Example LL. Evaluate the following integrals = _2sin 29-coro_ a | Bret pcding 6 —cos* 9-4 sing “de | ae +r vated) win J a . _ f _2sin2¢-coso Solution. (Let t= [oo enaang 22sin g cvs ¢)~ e038 2ssin A cos A=sin 2A . 79 “ sin? A+ cos? A=1 Asingeoso-c0s9 4, 5+sin?9~48ind © =J (4sin @- 1cos¢_ “d sin? g-4sing +5” 156 INTEGRAL CALCULUS MADE EASY Put sing=2z = coagdp=dz =f (4z-) (2? - 4245)" Let us write : 4z-l= a[Z @ ~ter5] eH = dz—-1L= 2-4) +p wD) = dz-1=2h2—4hop ‘Comparing the co-efficients of z and the constant terms, we have 4=% 3 ha? and -1=-4h+h = pa-1+4h=-144(2)=-14+8=7 Putting the values of A and 1 in (1), (dz — 1) 2 (22-4) +7 = = AD) Now Fe. =logifiotee] = log iat 4245) +c, (8) bz z= sing] = log Isin? 9-4 sin 6451 +0, 1 and 42 | Fae 1 ‘Add and subtract 4 to the denom. -f ——___ 2 *| fame (Jooemnor 2) 4 1 -J @-aF ei Put z-2=y = de=dy . L= Iz weg Funtire | tea ty 40, Dan @—9)-4¢ [e y=2-2) = tan (sin ¢-2)+ 0, wf4) fs ze sin ¢] From equation (2), T= 21, +7, = 2 flog |sin® 6-4 sin +51 + cy] +7 {tan (sin 6 2) + cq] [Using (3) and (4)) = Dog Isin? 6 —4 sin § +51 + 2c, +7 tan"! (sin §—2) + Te, INTEGRATION BY SUBSTITUTION—iI 157 =2 log Isin? 9-4 sing +51 + 7 tan (sin ¢-2) +c. where : ¢ = (2c, + 7e,). a 1 (ii) Let 1s Wawa (Wividing the numerator and the denominator by e*] Put etaz => -etdx=dz = e*dx=-dz e* 2 * 1-f 243e a Sma a) =f = _ OTS 843242)" Let us write : sea Zot eaee2] + > 2=N2z +3) +H A) => z=D24+3hep Comparing the co-efficients of z and the constant terms, we have tem = aed 2 1 3 and O=den = u=-sn=-a(3) ; Putting the values of 2. and u in (1), re p2e+9)-3 1 3 Far+9-3 ‘ a (c? +8242) 2z24+3 3 1 = Te al Fae +3242" a5] z 43242" = 1e-3h 43h wf) 22+3 fe | =| —- se .dx = log! he Now Ne) yam [ Peay Ge=eBlF@) +e a log Iz? +824 21 +0, slog te +e +21 +6, =log| alec fy zee 158 INTEGRAL CALCULUS MADE EASY 2c + nog] e321 (8) 1 and ae] Pesesa'@ : Add and subtract $ tothe denom. =f7 ade 2 2 9) 9 . G } =? {#ee+ } See » (Jeoemor) 8 1 = ide Ta 2) 4 3 Put (2+ 3)xy = dz=dy 1 . . a [Pr sing f y-(2 etl —4 | +e =log| 3-2 +04 atote Ie" +1 +0, =log | 1/2 + BoE eae | 7 lee’ ak +e Al 1+2e"} ? 4 From equation (2), 1’ 3 I=-gh+gh 1 1+ Be + 207 3 lte* , = 3 fie (222) oy] inl Bo} [Using (2) and (4)] 1 1+de*+2e*) 1 3 lte* ) 3 =— fog 2B) Be, Siog( Hey Be PE elec iad 2°" 14208) 2 at Gii)tet =f 9 ate 4d INTEGRATION BY SUBSTITUTION—II 159 [Divide the numerator by the denominator] s i -4z* +29- 5] de hag) a8 44x? Cx4 — 427 +20 8 44x =fxtide—4f tide - 1 — 4x8 + de® +20 J ..ce-90f arya een js 1 1 t+ Bs sphpdesttan at [ y wsing f xt +a" ‘| 20x? 20x” +80 8 gt - - = 2-2 4202-80.) tan 2 +e 5 3 = bat Se e20e- sotan? = +e, Example 12, Evaluate the following integrte i a 6x+7 an 2x4+3 olesaa-« wf ade x 5- 2x (id) de (iv) -dx. 6x+7 6x +7 Solution. (i) Let I= des fution. i) J la-5)e-4) Iptits 92420 | Let us write : d Gxt TaR. oo ie oe + 20] tH. > Gr +7 =A(2x-9) +p wf) = 6x47 = Dx +p Comparing the co-efficients of x and the constant terms, we have 6=2% > A=3 and T=-MM+p > w=T+9=7+98)= Putting the values of 2 and j1 in (1), Gc +7 =3(2r-9) + 34 12f 3(2x-9)+94 yx? 92420” 2x-9 1 “Trae “I To = 1231, +341, wl) 160 INTEGRAL CALCULUS MADE EASY t ol aoe x -9x+20" Put zaa*-9r420 = dz=2x-9 fevid = 2212 40, =[y-e- = 2yx* 9x +20 +e, (3) be z= x*- 9x +20] 1 and ne | Gate ad 1 2 81 [ee to the denom. + oey | 1 = log} z+ 27 -(2) | 2 9 9y 1 = log (=-$}+ (=-3] -7|te [ z-x-3] = log (=-3)+ a? axa +e A) 2. From equation (2), 1=31,+941, [Using (3) and (4)) (3) 9x +20 +a] =a(2 PO 29 +) +94) oe INTEGRATION BY SUBSTITUTION—4I 161 = 6 yx" - 92420 +34 log ( ~ 3) +192 +20 +e where: c= 3e, +34¢,. Gi Let T= f BA de Of frteace1 | Let us write : d ar+3s gg tee De = We Bade dey of) = Qx+3 =2ar+4hep Comparing the co-efficients of x and the constant terms, we have M=2 + Aal and Aheusd = ps3-4h53-4(1) Putting the values of and pin (1), (Qe 48) = (Qe 44)—1 f?ees yx? +4x+1 deed 1 = I Sew. ke - | Sn \ en 1, (2) Now y=] Berd Jerarel « Put oo xt4de¢1ez = (Qx4+4).de=dz 1 s uef[y-e dx = I =Qyx"+4xt+1 4c, 8) be zaxte dee 1 ah S—— h Sen and Add and subtract 4 to the denom. 1 2 dz 1 leas * (Jeoemors) = =J apes ty oF r+ 2y* - i)? Put x+2=2 > dy = dz 162 INTEGRAL CALCULUS MADE EASY owl are [amin Jeo ole feta? I me =log | 2+ 2-5 | +, log | (x +2) + fox 2*=3 | +6, fe zee+2) =log |r42)+ Jat vdeo | +e, aC) “From equation (2), I=1,-l = (24e +4e+1+0,}-(I0g| (e42)+ fa? 4241+ ¢2) (Using (8) and (4)] 22 fx eda+1-log|(24204 fx 44x41] +0. where:e=(e,-e)) (iit) Let “| Let us write: rah. 2642-944 = xeM1-2)4) D = wee they Comparing the coefficients of x and the constant terms, we have = = as} and OnAtp = penae-(- Putting the values of A and jt in (1), xs-2a-29+4 “ Te Vie mtg de “(era -x* 1 = Is-5h+ 5h =D) INTEGRATION BY SUBSTITUTION—I! 163 Now , Jes 1-22 y8+x- x Put 8+x-x=2 = (1-2r)dx=dz 1-f xpee feta =e +e, a24brx-2? +c, AB) [rs 288 4+x-2x7] 1 1 and = | sy cae = f ——— ° le leas 1 J 1 1 ode LD) fogs (5+) (= #4) Add and subiract + to the denom, 1 pod + (Seoorats} 1 |, (2e-1 sin + wA4) “ (a) & 1 ; Legh (Using (3) and (4)] (2 Brew +e) Bfan( zl) .1. fae-1) 1 = foe F Be sdaw(Me) de 164 INTEGRAL CALCULUS MADE EASY =~ Bee— + doin (Ba )+ where : (-Ze+3e) , 5-2e (iv) Let le Let us write: 5-200. £ 642-29 4n = B-2e=A(1-2x)+p wl) => Ba Qea-2arthep Comparing the co-efficients of x and the constant terms, we have =2=-2 = del and Bake = 265-225-124, Putting the values of 4 and pin (1), (5-2x)= 11-20) 44 if (L-22)44 V6+x~-2" vd 1-22 deta f = Tel +4, (2) Now u=f 3-25 y6+x-27 de Put 6+x=x2=z = (1=2r)dr=dz ef ete fetes =2J2 +0, = 26 +x-27 +0, 3) [z= (6 +2-x)] srl and Add and subtract ; to the denom. «(2 4 : (Jevett ot =) =} INTEGRATION BY SUBSTITUTION—IE 165 Pat (x-3}== = dende 1 , 1 * =) =P. & By using ae] Om yen 2 asin? 2 4e,esint = 46, 5/27 5 sin ner 1/2) te, axl sie = Jee AA) From equation (2), lel, +41, =(2Vere=2" +a}4[in = =2y6+x-x* +e, ¢anin™ (221) te, J «| [Using (3) and (4)] =2 Grew +4sin (21) +6 where :¢= (¢, + 4¢3) Example 13, Evaluate the following integrals : bx4+3 x+3 “ ods «i ods J et +4e+10 i arrer of xt? ay f (ae cio | ae (io) f JO=* ae 3 irs oy [ ant tbe te) f JE ae wid f Tre 72 it) fx re dz. Solution. (i) Let I= 5r+3 So ck Jx? +42 #10 Let us write : Sreden 2 itodrs 10)ep => Sr+3=Ar4 4) ep AL) = Se+3= 24 dep Comparing the co-efficients of x and the constant terms, we have B= = aoe and B=4A+p 3 p=3-423-4(3) 166 and INTEGRAL CALCULUS MADE EASY Putting the values of and yin (1), (6x +3) 5 2 (Qx+4)-7. Beare 4)-7 “ I=f 2 ax Ie +4x+10 a eee ee de 2) Se are10 Vet eax+ 10 a~ Thy 2r44 Now iJ Ga Put 2274+4¢4+10=2 = (Qc+4).dradz 1 wha 1 -v2 z o l= ede= fz? de +e, =2V2 +¢ Ie J wT, °° 1 2 2 fx? 440410 +c, (8) [z= G2 +404 10)] 1 = de & las 1 = «dx J Va? +are4)+00-4) ‘Add and subtract 4 to the denom. [! (Jevemors) =4 1 - Jere * Put r42=z2 3 dt=dz 1 . 1 al ae [prasine) ogee mlog|2+ Ve +6| +, a log |x +24 fz+2)7 +6 | +9 = log |2+2+ fet +4x+10| +0, aC) From equation (2), I 5 h- 71, INTEGRATION BY SUBSTITUTION—II 167 : HE yc +ax+ 30 +e1]-7[oe| (x +2) + yx? 4x4 10 | +2] (Using (3) and (4)] =5 fF +4e+ 10+ Se, Tog] (e+2)+ Ya? +424 10 | -70, 2B (a dey 10 ~Tlog| x +2) fa? +4410 + 0, where ¢ a+ Gi Let =| EA , ~4r-= Let us write : ae3eh. 2 5-4e-2414n = x+3ehC4-2c)+p wf) = x43=-2ar-dhep ‘Comparing the co-efficients of x and the constant terms, we have 12-24 => aed and =-4ty 3 wade nasea(-J\a1 Putting the values of & and p in (1), (e+B)=- 34 O41 Fa 2eret 2 iJ 5-42 -x? as (-4~- 2x) def V5-4x-27 1 v5 -4x-, = AQ) ‘Now Put S-dy-x2=z = (-4-2xdeade 1 de 1 . ¥ ef yerfere +e,=2 Ve +e, =2\5-4x-x* +e, AB [z= (6 -4e-23)] otf aoe 168 =f See V6 - (4x +2") =f Leen ve Vora-arer +a) -J 1 dx Yo-es a” Put x+2=z = dx=dz ke | ager -@ 3 4 (e+2 =sint |=] +69 From equation (2), = sin! ssin! 5 +e, te} eh Fla eae + 0/]- | INTEGRAL CALCULUS MADE EASY ‘Add and subtract 4 to the denom. 2 2 (Jovetto x) =4 sean to a [eel te wwf) [Using (3) and (4)] x+2 a- (5-42-27 ~ ferrin ($22) +0, x42 Let us write : esoer. Zee vep = et2ah xen - 5-42-27 sein (222) 6, where c = ata). ‘Comparing the co-efficients of x and the constant terms, we have tet = aed 2a => a2 Putting the values of 1. and pin (1), and s42e2 nee INTEGRATION BY SUBSTITUTION—I 169 wel) 2z+e 3) (vo 2 =G?-D) sag teh F | and nae _— A) From equation (2), = ; +2, = 3 [2veF= 146 ]+2ftoe| e+ Va? =1+e] [Using (3) and (4)] =e =+de,+2log|x+ya7=1| +2 ey = fF =1+ Blog| xf? =I +0, GuyLet T= f [Oo de (@-x)* te (sm (On rationalization} {a-)(a +b) = a? 64] (a? ~ x7) ° Spa fae) =al,-1, Now 170 and and INTEGRAL CALCULUS MADE EASY 1-f wor * Put a?-x?=2 3 -Urdeads = xdr=—5 ‘ neft +: (e)-He a (8) De z=a?=x3] 1. From equation (1), T=al,-l sa [sin Z| [sara +2, (Using (2) and (3)] sasin*= 400,+ fa? e a esint ey fa? x? 40 where : ¢ = (ae, —c,) wie sf ft ae ¥ =f ce de [Multiply and divided by (1 + x)] -f ase 4 ES wea) yore teed. Zest en Let us write: > leeek(Le deep A) = ltceDe+hen ‘Comparing the coefficients of x and the constant terms, we have 12% = asp Lahey 3 wal-aet-ga3. Putting the values of A and jin (1), Lesa plean ed, INTEGRATION BY SUBSTITUTION—II 171 = 2) Now Put 3) and Add and subtract Fto the denom. Ta i ey 3 ~f2 at (« teed} ‘ (Joan ox} 4 1 e- =log (+g}+ve oe +e, 4) .. From equation (2), le5 + } L (Using (8) and (4)] whe Blt (2eee rer} g fel (xed) area “| INTEGRAL CALCULUS MADE EASY (vi) Let = Isal,461, 5 x Ne l= wf are =i 4x° ite” = Jog st +e] +6, and = a= aa 1 “al w@ne* Put Baz = tdesdk z. From equation (1), Ieal+b ww) (Multiply and divided by 4] i) [: je. des lag ite] [Multiply and divided by 2] By using [ ade alien Zee x? +a" a a 3) [Using (2) and (3)} 1 Lent (2 sof iogtet14ey]+0] tn ‘(2)+4| a = Glog tet ctl oa Rian) 0, = Glatt eet « Bea (Z =| +4 where : ¢, = (ae, + be,) INTEGRATION BY SUBSTITUTION—II 173 z_ 3 . ja? x" (vii)Let_ T= [x J . de Ile Put faz = wdredz = xdzx {On rationalization} aly [@-2* ip _a?- apa =a) yar en) Goa bs @-b)(@a+b)=a?-84 and (8) bey sat— 24] 174 INTEGRAL CALCULUS MADE EASY +. From equation (1), [Using (2) and (3)] = 2 win E dere +e. ts z=x4) Beample 14. Bunluosh the fillewing integra of 4x4] de ‘ tin f Spt 8e3 5x43 etdr+2- an f 243 ta2 e241 yy eoxtl (Dividing numerator by the denominator) “ I= f(=+ 344) 4 eS eeaile xt ou +t =f xde+ 2ett oae 2+ ofrdeef oa [Py sin f ce. dx = log|f(2)1+.c in the second integral and f pig ee diel 52 + inthe tid integral 2 2 ? yy tog] 224 5 + log Ix’ U+5 log mi +c. Gi) Let Ie] apres dz (Dividing the numerator by the denominator] INTEGRATION BY SUBSTITUTION—It 175 | . 1=f ya 2etd | x?48x42) x? 46x43U1 x 43x42 x43x42 i = fides atl ee a xP 4 3x42 ! 2243-2 =[lide+f SA dk J | Peace | 2x 1 i = fides oO _1—. i fa de Ja35s Jann | f(x) ’ -dx =logi f(x)! | [ Ta) ogi fl+e > Tox 4 log Ix? + 3x + 21-21, +0, 0) ‘Add and subtract Fo the denom. Putting this value in equation (1), we have I=x+log Ix?432 +21 -2[te x41 x42 wales 176 INTEGRAL CALCULUS MADE EASY a2 + log Is? + 8¢ + 21 ~ 2og | =** zt = ey +0, ax +log Ix? 43x 421 - sig 2) whare : ¢ = (¢, —2e,). 2 Gi) Let = J SpE pGp- de Dividing the numerator by the denominator] . le ip- (6x43) fe). a 2? 462412) 2? G ¥46r+12 2462412 a frae-of 222 --- x +6r+12~ "12 2 fra By test 2e+4 x 4+6x+12° [Multiply and divided by 2 the integrand in second integral] =f tdc-af tS? ae [Note this step) x? +6xr412 = Jrdr-af sto aevef = 3. +654 12 #4 6x4 12 exe Blog 122+ 6+ 121 +6 f =. [: 0 are lagi reat+e] 1 = 2-Slog Ia? +6x4 12146 f Go ‘Add and subtract 9 to the denom. 2 : (Jevent ox} =9 1 =x—8 log Ix? + 6x 4121 +6 { —— ax = lore [ ~ haat Eso] a a =x-Slog la? 6x4 12146, 4 td, 3 V3 Sx Blog Ist 46x 121 + 2 tan 273 , 8 exf4 2041 ovtet t= [ 222 #2841 (Dividingthe mumeratorby the denominator] x-xtl INTEGRATION BY SUBSTITUTION—II 177 wx) ett? +2041 2 1 fierae a Joa xed Teas +i lee axel Barer - + = ata = ferrets z Foi: oe aed Qe-1 ~ 242 wfecevaf aro) 1 a - 4 - sot ea 2 ——3r-1 = St arely a) Now I= wae woe Set-atpen (r= Dep M2) = Be-1=Qhe-A4 pe Comparing the co-efficients of x and the constant terms, we have 32 = a=} 1 and -1a-At+p = wea- 123-1 =5 Putting the values of & and pin (2), (x1) = 2 @e-n+) 2 2 ef 28 p+} Eg ceel Bp _2x-2 1 L 3 -dv+5[—>—. “al xi-x+1 ds AB) Fe) ae - dx = log | f(x) | Now Foxe’ [- Fay log | f(x) “4 = log Ix? =x +11 +e, ac) 1 d 4=/=—— an % Soom ‘Add and subtract + tothe denom. 178 INTEGRAL CALCULUS MADE EASY Put From equation (3), 3 1 L-gh+gh = 3 fogiz? - Al 2 (Ben = 5 [leet xrtlvo}e Zan ae Ira] (Using (4) and (5)] = 3 logis? — 8 + itan! (21), 2 = plogls? 4+ 5 1 + Fe tan (B5}-36 . pioelat— e+ I+ tan? (3) +e where: ‘Substituting this value of I, in (1), we have atde| tye = rare Ziogiat 2414 Jetant 25 te. (w) Let ef tt a f4+5x-27 * Let us write : e1nn. £464 p > r41eM6—20) +4 A) = e4l=-2he+ Shep Comparing the co-efficient of x and the constant terms, we have 1s-2 => az and 1=5h4p = yet-sret-5(- Putting the values of 2 and pin equation (1), rete 36-3040 INTEGRATION BY SUBSTITUTION—II ~l 6-242 =f 2 2 “| ge * Ay 5-20 T 1 ieee) ace 1 7 > T=-Fh+5h 2) Now 5- 2x il aoe Put 445e-s%=2 = (5-2x)de=de 1 a parle ST 40,222 +0, -j41 2 =2J4+5x-x7 +e, AB) De =[pee [4+ 5x ~x* 1 “J Ineo z=(4+52-2%)) and ro ‘Add and subtract 22 tothe denom, 2 ’ (Jovem ot) -3 2 4 180 INTEGRAL CALCULUS MADE EASY 2. From equation (2), 1 2 5 [everse-= sai} Zfe (“#)-4| [Using (3) and (4)] 3.2. INTEGRATION OF SOME SPECIAL TYPES OF TRIGONOMETRIC FUNCTIONS 3.2.1, Integrals of the Form \ emaxtbees *®! abaex®J atbenx®] asinztbomzve Working Rule : 2tan = 1-tan? ~ Step L Put sins = ——2_ , cos x = 2. 14 tan? 5 1+tan? 5 Step IL Replace (1 + tan? 2/2) in the numerator by sec? 3 : Stop UL Put tan ¥ =2 eo that 3 s00? 5 de = de a az +bz4e Step IV. Evaluate the integral in the step (iti) by using methods discussed earlier in the Article 3.1.1. ‘This substitution reduces the integral in the form J asinx+bcosx 8:22. Integrals of the Form J csinx +deosx Working Rule : Stpe L. Write: Numerator = 4 [Differentiation of denominator] + . (Denominator) te, asing +b cosr=3 £ (esins +d co02)+ 4 csin ed cos 2, Step II. Obtain the values of A and j1 by equating the coefficients of sin x and cos x on both sides. Step IIL. The given integral aA eesin e+ dos x) - | Seardees Ot ude INTEGRATION BY SUBSTITUTION—I 181 Step IV. The second integral is ix and evaluate the first integral by substituting z=(csinx +d cos 2). 3.2.3, Integrals of the Form | seine beware Working Rule : Step I. Write : ‘Numerator = q (Differentiation of denominator) + p (Denominator) + r ie, Casings bem xsc)=g 2 dsinz seconesf+pdsine+ecoesp+r Step Il. Obtain the values of q and p by equating the coefficients of sin x and cos x and the constant terms on both sides. Step IL. Given integral da. (dsinx+ecosx+f) = | pdx + | @——_____ ar [ —__*___. rae] dsinx tecosx+f *S qaxecwnt de Step IV. The first integral is px. Evaluate the second integral by putting z=(d sin x4 cos x4 f) and the third integral by putting 2tan 5 1-tan? * * nd then taking z= tan 3. 2 SOME SOLVED EXAMPLES Example 15. Evaluate the following integrals : ; 1 « 1 | Tea | ar - 1 , 1 (ad f 1+ sin x +cos x ds tof 2+cosx" 1 . I »f—i_. a © | een © |S yyeinzecara Solution, (i) Let t= [ 1. de 446 sins 2tanZ Putting sin x = ——2-, we have 1+tan® 5 182 INTEGRAL CALCULUS MADE EASY -f 1 (1+ tant al 444 tan? <4 10tan= 444 tan? + 10tan > 444 tan? 5 +10tan 5 L+tan? 5 1 ne 5 2) Shea? Eesten 22" (e oeet A tanta) 2 2 Put tan 5=2 J gag? = gy * ee => get g d= de = sec? 5 dr= Ide 1 1 ” Is anes Tare Add and subtract 2 to the denom. dz 2 2) a 25 we (Zevettor 2) Pa I | co +e m/c INTEGRATION BY SUBSTITUTION—IL 183 1 1, 1773 1, |Qe+1 = slog) —2| 40221 a8) zee | 7°" 38 las ea| 7° tan * +1 = 5 10g) —2— | +e. 2tan S44 1 ié) Let t= f —+-- a © Prevesti 1-tan? = Putting cos x = ——+ |, we have 1+ tan? 5 {+ sec? A—tan® A = 1) 184 I i-_— Treinx + ovex 1-tan? = and cos x= ————2, we have 2 = dx 1+ tan? 5 1+ tan? = J ope 5 — 2% Fs t—tan? = = 1+tan? + 2tan £4 1-tan? 2 2+ tan 2 sec? = 2 -J = Put tan 5 == 2+ 2tan role dec? = dea 2 dee = geet deade m2 sec! 5 dem td 2 1=f sk (2d2) dx (+ sec? A- tan? A= 1) sfoie le J Seen tegietee] z+1 = log lz+1l +c =log| tan E41 +e (ev) Let 1=f os INTEGRATION BY SUBSTITUTION—II 185 -J—wie [+ see? A-tan? A= 1) 1 ax = == By de=4e = Janae [ yr osing | des Ft Ee] =2. otant( 4) +e = tan] 2], [ zetan> WB eT * . 2 (tet t= | ees 186 INTEGRAL CALCULUS MADE EASY 1+ tan? = van? ley (a +6)+(e—b)tan® 5 sec? = “ sf ——_—_2_ & (ts sec? A-tan?A = 1) (a+ 0)+(a-b)tan? 5 x Put tan =2 looxt ae = des => ze gure = sec? 5 de = 2de 1 1-2f @rb+e-58 © — =(a-0)) ape ab Now there may be following possibilities : Case 1: When a > b, then ate A 1 1 at ae [Prong f 2a ee hs a-x -22 +e 187 INTEGRATION B¥ SUBSTITUTION—I tog| Ee +vo=a-2| | ‘Vora=Jo-az 1 ore + Joa tans a2 5, = log yeaa Wore - fo-a tan = Case [1 : When a = 6, then 1 t= [ ——— —_ S aheax 7] Tracer 1+cos 2A = 2cos* A = 14 cos A = 2 00? & Z 2 cos’ on tan x. den (net? 2 2\+0 . 1 teilet I= | Soma reoee 2tan F 1-tan’ £) Putting sinx=|———2-| and ooox=|——2 |, we have 1+ tan? tan? = | 1+tan? * an? = J 2x = 22 + = 44tan=41-tan?= 343 tan’ tan=+1-tan Put tan 5 =z dec? ¥ dade = sec? * de = 2de 2 2 2 = 1 1 ” I= | ———_ . (dz) = |] =——, . Jaqgn Oa) Sasa ‘Add and subtract 1 to the denom. 1 2 so ef - S armas : (J everor =] =1 INTEGRAL CALCULUS MADE EASY 1 Jama Put z+isy = deady 1 1 xe d= tan”? Eel @ 1 . : -f— B . if ae [By sing J ore stanly+e te stantiz+ +e ~tao" (tan Z+1) +6 [: tan] 3 2 Example 16. Evaluate the following integrals : iy _tsina —itsins ay (9 | etrea y=@+D) -. I | ees Wumz ses I a [ cosecosx+ I —[—Sat dx Wt | Sos cos @ + cos x ar wf avbainx * 1 Solution.) Lat I= f ——— ge ae 2tan = 1-tan’ $ Putting sin x = | ———*> | and cos x = |—_——* |, we have ‘tan’ ode 2tan = + J3 - 8 tan? = sec? = ={ ——2—_« [> sec? A-tan?A= 1) 2tan 5 +03 ~ 8 tan? 5 1 x +e 1+3 tan= = Flog} ——2| +e, 2 3-VB tan = 7 1+sin x tet =f ain z(1+e08) 2tan= 1-tan’ 3 Putting sinx =| ——2—| and cosx= —— |, we have 1+ tan? = 1+tan? 5 [Brosiog f pox =k 1 a a 2a ‘Add and subtract 3 tothe denom. de 2 + (2 =i } : (Jevenor x} ; +x == 190 INTEGRAL CALCULUS MADE EASY dx [: sec? A- tan? A = i] Put tan J soc? => Ssec 2 =z x x gard > sec? 5 de = 2dz 2 2 Taf S42" (odeye f HEP ae 4z az a3 f(See+2)e “al Race fades jae] 2 =f betets +21] x0 [: J Lae=togizte<] 1 #|, tan? 2/2 x + getan® wn 822 stun [- eu] inte T= | aay x 2tan= Putting sins = ——2_, we have 1+tan? = INTEGRATION BY SUBSTITUTION—1! 191 1¢tan? 2 1+ tan® potten (sec? A-tan?A = 1) > geet F dead: = sec? drm ddr 1 * 1.) age Jaa ‘Add and subtract 4 to the denom. {=e 1 7 (2? -4244)4+1-4 o (Jeoerr or 2] 4 1 2f—t—a Soars Put z-2y = de=dy 1 . 5 wafpcigre [Brute] 5 =2 55 = 128 +e 1 2-2-J3 =k =2z-2] fs 8| 2-2 va |*° be yaaa tan = -2- 9 © = —= log] —2—_—_] +c. [ stan 3] Va S| tan 2418 a Goyer T= f SAKSBETT gy (08 0 +008 x cos at cos x + cos” of + 1— cos” a dx C08 0 + 008 [Add and subtract cos? «to the numerator.] cos @: (cos x + ¢os cx) + sin’ (cos a + cas x) [y cos? A+ sin? A = 1] 192 INTEGRAL CALCULUS MADE EASY 1608 0. (cos a + cos x) sin? ee | te (cos+ e082) (eos +c08 =) senso f t.dresin’ a f ——1 c08 & + COs x = Iscoso.x+sin? oI, of) 1 Now =| Saatome 2 = lttan’> 008 + cos a. tan? £4 1-tan® 3 L sec? A-tan?A = 1] x 2 1 “ 1,- f ——* ,__,, ae * | Sarwar’ ? L 1 2y5 martviren #] G3 cosa) Gana? & 1-cosa “y 14+e0s 2A =2cos® A _ 2 1g = 1+008 A=2ens? 4 © Bein? =" eos? # in? 2 2, 1—cos 24 = 2ain’ A Bein? S => 1-cos A= 2sin? > ch tog| S42 vel 2a | INTEGRATION BY SUBSTITUTION—1! 193 | a x - 1 cot 5 +tan 5 ve - a = | ent 5 —tan 5 © to = 1 cot $+ tan 5 ain Soma | oe 8ean|* 2°83 2°"3 cot & + tan = > 2sin A cos A=sin 2A “=a: —2— 2 J+ asin con xsin A cot tan = ein 5 ons 5 =sin (Let Putting see? A —tan?.A = 1] Put tan = pte = eect? deade + sect = den Ode Par) 2 L 1 2 “ tT sara a] ae) @ INTEGRAL CALCULUS MADE EASY Add and subtract & to the denom. a? 7 2 ae ey ey ne” » (Gevett or} =o a a a a Put 2 1 * a) alata Y + 2 Now the following possibilities are there : 2 Case 1: When a > b, then [: | 3 2 1 2; ae Te ale 8 INTEGRATION BY SUBSTITUTION 195 If Case II : When a < 6, then 2 1 1-2/ —1_, « yee Case III: When a = 6, then 2 a+bsinx 1 1 1 =f —*—a-+/ + _ J 4 a! avn [On rationalization] be (a+6)(a-8)= a? -87 atyptctins Ce sin? A +cos? A = 11 a! cos? x “ae =} tanx~seer)+e. }ae= 3 J ceetx sees tana de @ 196 ‘TWTEGRAL CALCULUS MADE EASY Example 17. Evaluate the following integrals : 1 1 © | Srmertame | Saeeez Solution, () Let1= [ ——*__ ae 2+cos xr-sinx 2tan = 1-tan? = ——2__ | and cos x = | —— |, we have = 1+ tan’ 1+ tan’ 2 if Putting sinx = ca) 2tanZ a| oom a |_| 2s 2x 2x L+tan? >) [i+tan?S 1+tan? = = cee pe en 242tan® = grin tan? 5 -2tan > 1+ tan? = ttan"S tan? 21am +3) sec? = =f a [+ sec? A-tan? A= 1) (tan Z-atan 343) 2 = Put fan <2 Loos > jee’ E dreds = sec? 5 dra dds 1 tJ Fae (2s) =2 J ary & eee | —_—__i1 2 =| Ee * ! (J eveteor) =1 1 =| anwar Put z-lsy => dz=dy INTEGRATION BY SUBSTITUTIONS 197 . 1 1 ax By using [ ———g dx =— tan? 2+ [» len a mn a 4 le y=@-) Fa1 wim (* fs eet] .. 1 (i) Let t=) sae 1-tan?= Putting cosx= — , we have 1+ tan’ 2 [y sec* A~ tan? A = 1) . 1 omer) 1 * t=] apg oe 2) ere ss(ja(Glee rae pein ee] ae [mm 198 INTEGRAL CALCULUS MADE EASY and Example 18. Evaluate the following integrals : 2sinx+3cosx Aain x45 cox © | Sasttews Oo |S inetdeces ; 2sinz+3c08x Solution. (i) Let I= | oot demm Let 1 and j be constants such that 2sinx+3.c08x=2% 3 sin x +4 cos x)4 1. (3 sin x + 4 cos x) a = 2 sin +3 cosx =A (3 cos x =4 sin x) + (3 sin x + 4 cos x) MA) = Qein x +3 coax = (Si — 4A) sin x + (4p + 92) cos x Comparing the co-efficients of sin x and cos x on both sides, we have 3p-40=2 -() 449223 (2) Multiplying (1) by 3 and (2) by 4, we get Su - 12 =6 (3) 16y + 12, = 12 aul) Adding (3) and (4) ; we get ps1 => ps 2 Using this value in (1) ; we get 3(8)-a.2 = 7 -224) 1 4 > fhegs 2 he G5 Equation (A) becomes : RI . 1 nas 8 i Zein +3 cosx= >; (Scosx—4sin x) + 5 (3 sin x + 4 cos x) 18 jo 1 . gg Gains + 4008 x) + 55 (Bcosx— 4sin x) “ 1s] (Basin x +4 cos x) -BfraeZ 3cosx— deine Ssin x + 4cos x = tt beg [Sain et deonel +6. [: me. = gts] " 4sin x+5 cosx tet t= | Sein rede Let 2 and u be constants such that sins +8 coo 2) = 4S sins + 4002) +n (6 sins + 4.009.) > (4 sin x +5 cos x)= 4.(5 cos x—4 sin x) +p (5 sin x +4 cos x) fA) => (4 sin x +5 cos x) = (5p — 4A) sin x + (4p) + 5A) cos x INTEGRATION BY SUBSTITUTION—II 199 Comparing the co-officients of sin x and cos x on both sides, we get Su 4ha 4 wl) and 4u4 5h 5 (2) Multiplying (2) by 4 and (1) by 5, we get - 20h = 20 ww(B) 16)1 + 202 = 20 4) Adding (3) and (4), we get 4lp=40 = p= * Using this value in (3), we get ‘40 1000 »($)- 20h = 20 => == -20 = 20h 180 > “ay 7208 => Reg Equation (A) becomes : (4 sinx +5 cosx)= (Ssin x +4.cos x) -Bfraed 5sinx+4cosx = I lee. -Berd log 15 sin x +4008 xl +6. Example 19, Evaluate the following integrals + aj a Solution (Let = [pee a rtya' a Put x=asin®@ = de=acos0d0 acos add n 12 {| —S——— ae | nae eteetens -J acos® ao asin @+ayi-sin?@ cos @ = ao f sin 6 +cos6 “ . 1 wo f a+bootx “f cos 8 sin 6+ eos” 6 20088 =2) gore Fino +eos0 @ 9 40... gy Ooms t— 4 sin x) + 77 (5 sins +4 cox) {Gain x+400n3)+ 9 (6cosx- Asin =) a One eos tg Ose tans f@ Seoax—4sine [: J f@a- logo 1+] f@) dx, [. sin? A + eos? A = 1) [Multiply and divided by 2] and INTEGRAL CALCULUS MADE EASY 17 cos 8+ c086 = 2/ sind+cos0 Af (sin 0 + cos @) +(cos 6- sin © a “2 (sin @ +cos 6) {Add and subtract sin 8 to the numerator) 1 1 cos@-sin@ fe) 5) bao+s ree? [: Fat =ogiscnie<] . 2g < x=asin@ = —=sin® at a =38t5 digg isin 8 + c08 61 +e ax = @=sin = ‘! cos@=J1-sin? 6 =f—s =) Gein x+ bcos Let A and j1 be constants such that sina #2 S (a sin +b cos2)+ wa sins +b e092) = sin x = Ma cos x—b sin x) + pla sin x + b cos x) AD => sin.x = (ap— bh) sin x + (bp +a) cos x Comparing the co-efficients of sin x and cos x on both sides, we get ap-bh=1 (1) bu-ak=0 (2) Solving (1) and (2), we have HLA 1 a” -b at4e a = bear md hee Using these values in (A), we get (asin x + 6 cos x) bees x- bain) + 4 a? +57 at aby INTEGRATION BY SUBSTITUTION—II 201 \ gig pe tose bain d+ a (asin x+ bcos 2) (asin z +6 c0s2) a ah f seen bsins . f P& “a0! asinstbows* o aap] de [: lie logis) re] log Ia sit b a Jog 1a sin x + bcos x1 + ate = ata e Example 20. Evaluate the following integrals sins -. of see wo f de, Solution. (i) Let I= | —""*— dy sin x- cos 1p _ 2sinx - 23) wea (Multiply and divided by 21 1p sine LA singesins 2/ sinz—cosx (cos x + sin x}+ (sin x — cos x) = tain de 2 (sin x —cos x) [Add and subtract cos x to the numerator] i 1 eee det Sf tad [: fj. de= lage] 2/ sinx-cos x f(x) 1 ., 1 #9 leg |sin x - cos 1 +5xte. 2 (Let I= wt 2 5 mony de [Multiply and divided by 2) iL cos x +cos x “24 cosxtsinx 1 ¢ (cos x +gin x) +(— sin x+cos x) fj —STrer——v er 2 cos z+ sin x (Add and subtract sin x to the numerator] zt Af Cin +009 x) ef £2 dee togl oire| ag) hers cores & [ Fits) Bf Fa 2 tog eos x +sin xl +0 a** gis " INTEGRAL CALCULUS MADE EASY Example 21. Evaluate the following integrals : io [ ozteainasd cusxt+2sinx+3 4 wf Scoz+6 ge deosx+ Ssinx+6 2cosx+sinx+3 win f —2eezt2 _ Beoeet 2 ivy f St Peet deine sinxt2cosx+3 2sinx+cosx+d mn cos x+2sinx+3 Solution. () Let = [ f= *EE=*S de Let p, g, r be the constants, such that cos x+2sinx +3 =pl4cosx+5sinx+6)+q SZ toeonxsSsinzs Ber > coer 2sinr +3 = p(Acosx+5sinx +6) +q(-4sinx + 5cosx)+r (1) > coez +2 eine +3 (4p + 5q) coe x + (Gp —4q) sin 4 6p +r Comparing the coefficients of cos x, sin x and the constant terms, we have 4p-+5q=1 5p-4q =2 6p+r=3 Multiplying (2) by 4 and (3) by 5 and adding, we have 4 4ip=l4 => pay ‘Using this value of p in (2), we get 14 56 4(38) +5021 = oq=1-F5 —5_ 3 = 4*-5xal * 94-g Now using the value of p in (4) ; we get 4 84 6(F)+r=3 art Ba ed ‘Substituting values of p, g and r in (1), we get corse 2sine +d= M toooses Sein e+ 6)+ (32 8) (-asine + Seo0)s & a i 218 ae+( (Geos x +5sin +6) ) G4sin x + Sco8 2) 4cosx+5sin x+6 41 L 4cosx+5sinx+6 re) [: Fee) dz= lagiFo1+e] 1 Fouztbsinzr6 (5) = ets ~ Fi log 14 coo x +B sin +61 + 38 1 4cosx+5sinx+6 INTEGRATION BY SUBSTITUTION—i 1-tan? = 2tan= Putting cosx=——— and sinx= 2x x 1+tan® = 1+ tan? = +tan? 5 + tan? 5 1 J 1-tan? = 2tan* 1+tan? = 1+ tan? = tno tam 1+tan? = ax -[—_"2 dx 2tan® = +10 tan = +10 x Put tan 5 =2 lox x = Geectidreds = sect 5 dr=2de 1 27452465 Janes (ade = f , we have [fv sec? A- tan? A= 1] ‘Add and subtract tothe denom. afi y 25 : (Joverors} =2 INTEGRAL CALCULUS MADE EASY xB tan = 4 1 a* 3 [: = 4 = Joy +e ° g=tan | brie 2 fants 1, [zens 37 8- vB) =e ——_2—___l+e (6) 5") 2tan = gt 6+) By using (5) and (6), we get 2tan = +(5- | ui 2 T= 37 *- Gq lee Hd cos x +5 sins +61 + wR a1 41 2tanS yh 6+5)| - . cos x +6 Gi Lat t= | goetesss Let p, g, r be the constants, such that 5 con x+6 = pl2cos x+sinz+8)+q. Zccsresine +3) +7 = 5 cos x +6 =p(2cosx+ sina +9) +q(-2sinx + cosa)+r (1) = 5 cos x +6 = (2p + q) cosx + (p— 2g) sinx + 3p +r Comparing the co-efficients of cos x, sin x and the constant terms, we have Qp+q=5 p-2g50 Sp+r=6 From(9); p-29=0 = p=2%y From(2): Qptg=5 = 22g+¢ = 4g+q=5 => g=1 and: p=2 From(4): Sp +r=6 = BQ)+r=6 = rs6-6 = r=0 ‘Substituting values of p, g and 7 in (1), we get 5 cos.x +6 = 22 cos x + sin x +3) + 1(-2 sin x + cos x) +0 I= 22 [ Geestenee ‘Qeosx+sin x +3) + (—2sin x + cos x) (2cos x + sin x +38) (2cos x +sin x +3) [: j FC) ae ltt] fle) = [= 2 + log I2 cosx+sinx +31 +0. - Bcosr+2 wot 1) See Let p, q, r be the constants, auch that a 3cosx+2=p(sinx +2cosx+3)+q. (sins +2eosx+3)er INTEGRATION BY SUBSTITUTION—II 205 = 300s. ¢ +2=plsin x +2 cos x +3)+ (cos x—2 sin x) +r = 3 cos. +2=(2p + q)cosx + (p-2q)sin x + Sp +r. Comparing the coefficients of cos x, sin x and the constant terms : 2p+eq=s p-2q=0 Sper=2 On solving (2), (3) and (4), we get oo gad pa’ pegedes rez Substituting values of p,q and r in (1), we get Beoss +2 ¢ (sins +2.c0s2+8)+ # (eosx—2sinz)- 2 6 ¢ (sinx +2e08 x+3) 3 cos x-2sinx * 195) (anavBesa+) 5! cing+Beezes * -3{ 4 54 (sinx+2eosx+3) 1 2 S43 togisins +2.cosx +81 — ~(D) (2) 58 in x+ 20s x +3) fe) 1 Met = | epeersd 1+ tan? = Soe F leet) 2tan 5 42-2 tan’ E4a(1+tan 2) sec? = «f— 2 dz [> sec? A=tan® A = 1) INTEGRAL CALCULUS MADE EASY ld Beare rer; 1 "Add and subtract 1 to the denom. 22) aes ® 1 2 +224 D+6-D : ( coentof x) =1 1 hf 2422465 1 2f ape ™ Put @+D=y = dzady “ l= 1 * i 1 1% iL, B: aged é 2f yo” [ y using | ory ge= tn i] =2.4tan3(2 #2. Stan (Z)+ star! (224) +6 bi: y=@+D) x = tan? att +e 6) |v ze tan = 2 . 2. By using (5) and (6), we get tan = +1 8), - 2 te S243 tog tsin es zene at + tan +e 5.6 5 2 £ Say 6 3 . 8 2 Bits leg lsins + Boos +31 — 5 tan | —2— |e, . 3+ 2cosx+4sinz (io) Let 1s | Gane 2sin r+cosx+3 Let p, g, r be the constants, such that 8 +2e08z+4sing)=pQsins +0024 8) +g 2 Baines cnsx+ 3)+r = (8+ 2cosx +4 sin x)= p(2 sinx + cos x + 3) + 9(2 cos x—sinz) +r ow) => (8+ 2c08x +4 sin x) = (2p— gq) sin x 4(p + 2g) cose +3p4r Comparing the co-efficients of cos x, sin x and the constant terms, we have 2p-q=4 wn(2) pried AB) Sp+ra3 (A) INTEGRATION BY SUBSTITUTION—II 207 ‘Multiplying (2) by 2 and (3) by 1 and adding, we got 5p =10 => p=2 From (2):22)-q=4 = 4-q=4 = g=0 From (4):92)+7=8 = r=-3 Substituting the values of p, q and r in (1), we get (3 +2 cos x +4 sinx) = 22 sin x + cos. + 3) + O(2 cos x— sin x)~3 @sinx+cosz+3) 4, 4 1-2) Giinsress+y*) Ganveoorry & 1 3S aaeand 1 2 tte 8 ae eae 0 1 Let 2Qsin e+ coax +3 2tan 1-tan? = Putting sin x= > and cos x = ———®,, we have 1+ tan? = +tan? = 2 2 1 uel atan® | [1-ton?*) _ —., + — +3 1+tan? =} | 1+tan?= vtan'g) Gams 1+ tan? = -[|——_ "2 __. 244-tan? = 2% 4tang tt tan gt otdten 2 sec? = -[/——2 _. de [> sec? A-tan? A= 1] 2tan* S +4tan 5 +4 x Put tan Faz 3 oct = * = gtectSdemde => sect 5 dx 2de 1 l= | ———_ (2a) 7 Javan , J + 242242 ‘Add and subtract 1 to the denom. 1 -| —_—_ 2 Jz 72+ De se (J cvotr ot =) =1 INTEGRAL CALCULUS MADE EASY 1 = kJ ayaa Put z+l=y = dz=dy 1 5 1 1 a2 4-| ar? [arin Far ee ‘Eee star (2) +6 stant@+i+e tv ys@+D) = 1,>tan (tun 341) +0 (6) [ | By using (5) and (6), we get Te 2e—a tan (tan +1) +6, § PRR 10. MW 12. 138. EXERCISE FOR PRACTICE Integrate the following functions w-r.t. x Q. (1—10). = Beta + 2% Gi) Peeve 2 oe wo itt , 1-cotz ay o 1+ cotx Oa (i) xx +2 (ii) sin3 (2e + 1) (Wain 4x cos Tx Die + WPF 1 1 ote i) Sees 41 ae 1 Or, OO) eas 2 @ eer Ww 2 es ) (sec? x tan x (ii) see x log (see x + tan x) @ — (i) Fy oos ve. Evaluate the following integrals Q. (11-30) e ofa wf = 1 ar las © | sae of ee (i) JP coe ets [x= cos x

You might also like